nm update course ii: inflammatory myopathies/myotonic

41
1 NM Update Course II: Inflammatory Myopathies/Myotonic Disorders Richard J. Barohn, MD Steven A. Greenberg, MD Bassam A. Bassam, MD Bakri H. Elsheikh, MBBS, MRCP Annabel K. Wang, MD Gregory T. Carter, MD, MS Andrew W. Tarulli, MD AANEM 59th Annual Meeting Orlando, Florida Copyright © September 2012 American Association of Neuromuscular & Electrodiagnostic Medicine 2621 Superior Drive NW Rochester, MN 55901 Printed by Johnson Printing Company, Inc.

Upload: others

Post on 16-Oct-2021

4 views

Category:

Documents


0 download

TRANSCRIPT

Page 1: NM Update Course II: Inflammatory Myopathies/Myotonic

1

NM Update Course II: Inflammatory Myopathies/Myotonic Disorders

Richard J. Barohn, MDSteven A. Greenberg, MDBassam A. Bassam, MD

Bakri H. Elsheikh, MBBS, MRCPAnnabel K. Wang, MD

Gregory T. Carter, MD, MSAndrew W. Tarulli, MD

AANEM 59th Annual MeetingOrlando, Florida

Copyright © September 2012American Association of Neuromuscular

& Electrodiagnostic Medicine2621 Superior Drive NW

Rochester, MN 55901

Printed by Johnson Printing Company, Inc.

Page 2: NM Update Course II: Inflammatory Myopathies/Myotonic

2

Please be aware that some of the medical devices or pharmaceuticals discussed in this handout may not be cleared by the FDA or cleared by the FDA for the specific use described by the authors and are “off-label” (i.e., a use not described on the product’s label). “Off-label” devices or pharmaceuticals may be used if, in the judgment of the treating physician, such use is medically indicated to treat a patient’s condition. Information regarding the FDA clearance status of a particular device or pharmaceutical may be obtained by reading the product’s package labeling, by contacting a sales representative or legal counsel of the manufacturer of the device or pharmaceutical, or by contacting the FDA at 1-800-638-2041.

Page 3: NM Update Course II: Inflammatory Myopathies/Myotonic

3

No one involved in the planning of this CME activity had any relevant financial relationships to disclose. Dr. Barohn is involved in Speaker’s Bureaus with Genzyme and Grifols and is on Advisory Boards for both MedImmune and Novartis.

Any conflict of interest has been resolved according to ACCME standards.Dr. Greenberg is on the advisory boards for MedImmune and Novartis and has sponsored research from MedImmune. Any conflict of

interest has been resolved according to ACCME standards.

Chair: Dianna Quan, MD

The ideas and opinions expressed in this publication are solely those of the specific authors and do not necessarily represent those of the AANEM.

Table of Contents

Course Committees & Course Objectives 4

Faculty 5

Therapy for Inflammatory Myopathies 7 Richard J. Barohn, MD

Inflammatory Myopathies 15 Steven A. Greenberg, MD

Myotonic Dystrophies 21 Bassam A. Bassam, MD

Nondystrophic Myotonic Disorders 27 Bakri Elsheikh, MBBS

Neuromuscular Vignettes 33 Annabel K. Wang, MD Gregory T Carter, MD, MS Andrew W. Tarulli, MD

NM Update Course II: Inflammatory Myopathies/Myotonic Disorders

Page 4: NM Update Course II: Inflammatory Myopathies/Myotonic

4

Marcy C. Schlinger, DOBath, MI

Nizar Souayah, MDWestfield, NJ

Benjamin S. Warfel, II, MDLancaster, PA

Shawn J. Bird, MD, ChairPhiladelphia, PA

Lawrence W. Frank, MDElmhurst, IL

Taylor B. Harrison, MDAtlanta, GA

Shashi B. Kumar, MDTacoma, WA

A. Arturo Leis, MDJackson, MS

2011-2012 Course Committee

2011-2012 AANEM President

John C. Kincaid, MDIndianapolis, IN

Objectives - Participants will acquire skills to (1) utilize a pattern recognition approach elucidated through clinical vignettes in the diagnosis and management of patients with inflammatory myopathies and myotonic disorders, and (2) practice the vignette-based format used for many questions on the NM medicine board examination.Target Audience:• Neurologists, physical medicine and rehabilitation and other physicians interested in neuromuscular and electrodiagnostic medicine • Health care professionals involved in the management of patients with neuromuscular diseases• Researchers who are actively involved in the neuromuscular and/or electrodiagnostic researchAccreditation Statement - The AANEM is accredited by the Accreditation Council for Continuing Medical Education to provide continuing medical education (CME) for physicians. CME Credit - The AANEM designates this live activity for a maximum of 3.25 AMA PRA Category 1 CreditsTM. If purchased, the AANEM designates this enduring material for a maximum of 5.75 AMA PRA Category 1 CreditsTM. This educational event is approved as an Accredited Group Learning Activity under Section 1 of the Framework of Continuing Professional Development (CPD) options for the Maintenance of Certification Program of the Royal College of Physicians and Surgeons of Canada. Physicians should claim only the credit commensurate with the extent of their participation in the activity. CME for this course is available 10/2012 - 10/2015.CEUs Credit - The AANEM has designated this live activity for a maximum of 3.25 AANEM CEUs. If purchased, the AANEM designates this enduring material for a maximum of 5.75 CEUs.

Objectives

Page 5: NM Update Course II: Inflammatory Myopathies/Myotonic

5

Richard Barohn, MDDepartment of NeurologyUniversity of Kansas Medical CenterKansas City, Kansas

Dr. Richard Barohn is chair of the Department of Neurology at the University of Kansas Medical Center (KUMC). He received his medical degree from the University of Missouri-Kansas City School of Medicine and completed a neurology residency at Wilford Hall United States Air Force Medical Center and a neuromuscular fellowship at Ohio State University. Before joining KUMC, he developed and directed the clinical neurophysiology training program at the University of Texas Southwestern. Dr. Barohn is the author of more than 160 journal publications, 260 abstracts, and 40 book chapters on neuromuscular disease. He has served on the editorial boards for Neurology, Muscle & Nerve and the Journal of Clinical Neuromuscular Disorders. He currently serves on the Guillain-Barré Syndrome Foundation International and has served on the medical advisory boards for the Myasthenia Gravis Foundation of America and The Myositis Association. He also serves on the executive committee of the national ALS Research Group.

Steven A Greenberg, MDDepartment of NeurologyHarvard Medical SchoolBoston, Massachusetts

Dr. Greenberg is a clinical neurologist specializing in neuromuscular disorders. He completed his medical degree at Harvard Medical School and a neurology residency and neuromuscular disease fellowship at the University of California, San Francisco. He finished a second fellowship in bioinformatics at the Harvard-Children’s Hospital Informatics Program. Dr. Greenberg is an associate professor of neurology at Harvard Medical School and on the neurology faculty at Brigham and Women’s Hospital. He is a faculty member of the Children’s Hospital Informatics Program, where his laboratory is located. His major research interests are understanding mechanisms and developing therapies for autoimmune muscle diseases. He also studies how citation patterns influence scientific belief, research misconduct, and neurological disorders associated with zinc exposure and copper deficiency.

Bassam A. Bassam, MDNeuromuscular Program and EMG LaboratoryUniversity of South AlabamaMobile, Alabama

Dr. Bassam completed residency training in neurology and a fellowship in neuromuscular disease at Wayne State University, with additional fellowship training at Mayo Clinic. He is board certified by the American Board of Psychiatry and Neurology and the American Board of Electrodiagnostic Medicine (ABEM) and is a diplomate in the neuromuscular medicine sub-specialty. Dr. Bassam has served on various AANEM committees, including chair of the Workshop Committee; he current serves on the ABEM Examination Committee. His academic interests and achievements focus on neuromuscular disorders and electromyography.

Bakri Elsheikh, MBBS, FRCP (Edin)Neuromuscular DivisionOhio State UniversityColumbus, Ohio

Dr. Elsheikh is associate professor of clinical neurology at The Ohio State University (OSU) Wexner Medical Center, where he serves as the director of the EMG laboratory, co-director of the Myasthenia Gravis Clinic and education program coordinator for the neuromuscular medicine and clinical neurophysiology (electromyography emphasis) fellowships. He received his medical degree from the University of Khartoum, Sudan and his postgraduate internal medicine degree from the Royal College of Physicians in the United Kingdom. His neurology residency and neurophysiology/neuromuscular fellowship was completed at OSU Medical Center. He is board certified in neurology, clinical neurophysiology, electrodiagnostic medicine, and neuromuscular medicine. His research interests include muscular dystrophy, spinal muscular atrophy, myasthenia gravis, and other neuromuscular diseases.

NM Update Course II: Inflammatory Myopathies/Myotonic Disorders

Faculty

Page 6: NM Update Course II: Inflammatory Myopathies/Myotonic

6

Annabel K. Wang, MDDepartment of NeurologyUniversity of California-IrvineOrange, California

Dr. Wang is an associate professor in the Department of Neurology at the University of California-Irvine, the director of its Neuromuscular Diagnostic Laboratory and Neurologist at the VA Long Beach Healthcare System, Long Beach CA. She received her medical degree from McGill University and completed her neurology residency at Tufts New England Medical Center. She has an EMG and neuromuscular fellowship from Beth Israel Hospital and peripheral nerve diseases and neurology research fellowships from Mayo Graduate School of Medicine. Her research interests are peripheral neuropathies, autonomic neuropathies, and myasthenia gravis.

Gregory T. Carter, MD, MSDepartment of Clinical Neurosciences Providence Medical GroupOlympia, Washington

Dr. Carter is medical director of the Muscular Dystrophy Association Regional Neuromuscular Center and the Providence Hospice and Palliative Care program in Olympia, WA. After receiving his medical degree from Loyola University, Carter completed a PMR internship and residency and neuromuscular disease (NMD) fellowship at the University of California, Davis. He completed a pain clinical fellowship and a geriatric medicine mini-fellowship at the University of Washington. His current research focuses on improving rehabilitation management of patients with NMD. He has co-authored more than 150 peer-reviewed publications, 10 book chapters, and dozens of editorials, commentaries, and reviews and serves as senior associate editor for Muscle & Nerve. Dr. Carter is the recipient of the 2012 AANEM Distinguished Researcher Award.

Andrew Tarulli, MDDepartment of NeurologyBeth Israel Deaconess Medical CenterBoston, MA

Dr. Tarulli is assistant professor of Neurology at Harvard Medical School and directs the Neuromuscular Medicine fellowship at Beth Israel Deaconess Medical Center in Boston. He also is the associate residency program director and associate medical student clerkship director at Beth Israel Deaconess. Dr. Tarulli received his medical degree from New York University and completed his neurology residency and clinical neurophysiology fellowship at Beth Israel Deaconess. He has authored 23 publications, including Neurology: A Clinician’s Approach, a textbook designed for junior neurology residents. He won an AANEM President’s Research Initiative Award in 2008 and has a strong interest in clinical neuromuscular disease, clinical research, and education.

Page 7: NM Update Course II: Inflammatory Myopathies/Myotonic

7

NEUROMUSCULAR UPDATE II

Therapy for Inflammatory Myopathies

Richard J. Barohn, MDChair of the Department of NeurologyUniversity of Kansas Medical Center

Kansas City, Kansas

Immunosuppressive therapy is the mainstay of treatment in patients with active disease related to dermatomyositis (DM), polymyositis (PM), and necrotizing myopathy (NM)1 (Table). Autoimmune NM is often more resistant to immunosuppressive therapy than DM and PM, particularly if there is an underlying malignancy or a statin trigger. The overwhelming majority (23/25) of statin-associated necrotizing autoimmune myopathy (SANAM) cases required more than one immunosuppressive agent with relapse in 12 cases follow-ing immunosuppressive therapy tapering.2 However as in DM and PM, immunosuppressants such as prednisone in combination with methotrexate (MTX) or azathioprine (AZA) are the mainstay of treatment in autoimmune NM. For resistant or severe cases, adding intravenous immunoglobulin (IVIg) may be helpful. Third-line drugs include mycophenolate mofetil, cyclosporine, tacrolimus, rituximab, etanercept, and cyclophosphamide.

The few published randomized controlled trials of immunosuppres-sion in DM or PM compared placebo to AZA,3 plasma exchange,4 or IVIg.5 In addition, randomized controlled trials compared MTX with AZA,6 cyclosporine with MTX,7 and IV MTX with oral MTX plus AZA.3,6-8 The only positive-controlled trials are a small cross-over study of IVIg in DM5 and a randomized, double-blind, placebo-controlled trial of etanercept (50 mg subcutaneously weekly) for 52 weeks in 16 DM subjects.9

CORTICOSTEROIDS

While no controlled trial has been performed using corticosteroids (CS), there is general agreement that these are effective in DM, PM, and NM. Corticosteroids can be used in a wide range of regimens

and routes of administration. Prednisone 1 mg/kg/d (60-100 mg) can be administered for 4 weeks followed by an abrupt or tapered conversion to an every other day schedule. This taper is slower in patients with severe disease. A daily CS schedule is necessary in well-controlled hypertensive or non-brittle diabetic patients. While most patients feel immediately better after taking CS, strength im-provement is delayed by 2-3 months after the onset of treatment. An immediate response may suggest an alternate diagnosis such as poly-myalgia rheumatica. For the first 3 months, the typical adult patient remains on prednisone 60-100 mg every other day or its equivalent. If no improvement is noted after 3-6 months, or if weakness reoc-curs during the taper, a second-line immunosuppressive agent such as AZA, MTX or IVIg can be started. These treatments are initiated early on with CS therapy in patients with uncontrolled hypertension, diabetes, osteoporosis, or obesity and in those with baseline severe weakness. For good responders, a taper by 20 mg/month until 40 mg every other day, then by 10 mg/month, will reduce the prednisone dose to 20 mg every other day after 6-8 months from the initiation of therapy. After that, the taper is by 5 mg, and the interval is every 3 months to reach the minimal effective dose. In severe cases, the author prefers starting with a 5-day IV pulse methyprednisolone therapy followed by high-dose oral prednisone in combination with a second-line drug.

Recently, a randomized multicenter double-blind clinical trial com-pared oral dexamethasone pulse therapy to daily prednisolone in 62 patients with subacute onset myositis.10 The pulsed regimen consisted of dexamethasone given as six cycles of 40 mg/day for 4 consecutive days at 28-day intervals. While pulsed high-dose oral dexamethasone was not found to be superior to daily prednisolone as

Page 8: NM Update Course II: Inflammatory Myopathies/Myotonic

8

first-line treatment of idiopathic inflammatory myopathies, it caused substantially fewer side effects. Ten patients (33%) treated with pred-nisolone and one patient (4%) treated with pulsed dexamethasone developed diabetes mellitus. Mood changes occurred in 20 patients (67%) and eight (29%), respectively. Treatment adjustments for co-morbid conditions (hypertension, diabetes mellitus) were needed in one patient in the dexamethasone group, compared with 15 patients in the prednisolone group. However, there was a large number of early disconstinuations in both groups (21 and 17, respectively) for a variety of reasons.

Because the risks of longterm CS therapy are numerous, discuss-ing those with the patient as well as establishing a monitoring plan in collaboration with the primary care physician is integral to the management plan. Before CS initiation, a purified protein derivative skin test can identify the need for isoniazid in previously exposed cases. As CS is started, a baseline bone dual-energy X-ray absorp-tiometry scan is obtained and the patient is requested to seek an opthtalmologic examination, with yearly followup for both. Patients are maintained on oral calcium 500-600 mg two to three times daily with vitamin D 400 IU daily. In juvenile DM (JDM), the incidence of vertebral fracture 12 months following steroid initiation was 6% and these were mostly asymptomatic.11 The risk of fracture increased with higher steroid dose, and in the first 6 months with greater in-creases in body mass index or greater declines in spine Z scores.

Patients and their families are asked to be alert about personality changes and psychiatric side effects. Patients are also directed to reduce the salt and carbohydrate in their diet and visit regularly with the primary care physician for blood pressure, serum glucose and po-tassium, and measurements. The author advocates the pneumococcal vaccine and yearly flu shots. Given the immunosuppressed state, evi-dence indicates that seroprotection of the influenza A/H1N1 vaccine is significantly reduced DM patients compared to control subjects.12 In a retrospective study of 279 PM/DM cases over 15 years, 37% were admitted for a severe pyogenic infection (n = 71) mostly due to aspiration pneumonia or for an opportunistic infection (n = 33).13 There are currently no consensus criteria to identify patients who are at high risk for Pneumocystis carinii pneumonia (PCP) infection and would therefore benefit from prophylaxis. Patients with total lym-phocyte counts < 800/μL and/or CD4 lymphocyte counts < 200-400/μL are likely to benefit from PCP prophylaxis prior to initiation of and in the course of immunosuppressive therapy.14

METHOTREXATE

Methotrexate, an antifolate that inhibits lymphocyte proliferation, is an effective more rapidly acting second-line steroid-sparing im-munosuppressant. Oral MTX can be started at 7.5 mg/week, and in 2 weeks increased to 15 mg/week in two divided doses. The dose is then increased by 2.5 mg/week at 3 months, depending on response

Therapy Route Dose Side effects

Azathioprine By mouth 2-3 mg/kg/day; single a.m.dose Flu-like illness, hepatoxicity, pancreatitis, leucopenia, macrocytosis, neoplasia, infection, teratogenicity

Chlorambucil By mouth 4-6 mg/day, single a.m. doseBone marrow suppression, hepatoxicity, neoplasia, infertility, teratogenicity, infection

By mouth 1.5-2 mg/kg/day; single a.m. dose Bone marrow suppression, infertility, hemorrhagic cystitis, alopecia, infections, neoplasia, teratogenicity

IV 1 gm/m2 Same as by mouth (although more severe), and nausea/vomiting, alopecia

Cyclosporine By mouth 4-6 mg/kg/day, split into two daily doses Nephrotoxicity, hypertension, infection, hepatoxicity, hirsutism, tremor, gum hyperplasia, teratogenicity

Intravenous immunoglobulin

IV 2 gm/kg over 2-5 days; then every 4-8 weeks as needed

Hypotension, arrhythmia, diaphoresis, flushing, nephrotoxicity, headache, nausea, aseptic meningitis, anaphylaxis, stroke

By mouth 7.5-20 mg weekly, single or divided doses; 1 day a week dosing

Hepatoxicity, pulmonary fibrosis, infection, neoplasia, infertility, leucopenia, alopecia, gastric irritation, stomatitis, teratogenicity

IV/IM 20-50 mg weekly; 1 day a week dosing Same as by mouth

Methylprednisone IV 1 gm in 100 ml/normal saline over 1-2 hours, daily or every other day for 2-6 doses

Arrhythmia, flushing, dysgeusia, anxiety, insomnia, fluid and weight gain, hyperglycemia, hypokalemia, infection

Mycophenylate mofetil By mouth 1-1.5 gm twice a day

Myelosuppression, GI (diarrhea, nausea, abdominal pain), peripheral edema, fever, infection, opportunistic infection, malignancy, teratogenicicity

Prednisone By mouth 60-100 mg/day for 2-4 weeks, then 100 mg every other day; single a.m. dose

Hypertension, fluid and weight gain, hyperglycemia, hypokalemia, cataracts, gastric irritation, osteoporosis, infection, aseptic femoral necrosis

Cyclophosphamide

Methotrexate

Rituximab IV Two doses of 750 mg/m2 administered 2 weeks apart

Tacrolimus By mouth 0.1-0.2 mg/kg/day split into two daily dosesNephrotoxocity, GI (diarrhea, abdominal pain), hypertension, electrolyte imbalance, tremor, infection, hepatotoxicity, teratogenicity

Mild infusion-related adverse events (headache, nausea, chills, hypotension), anaphylaxis, infection

Monitor

Monthly blood count, liver enzymes

Monthly blood count, liver enzymes

Monthly blood count, urinalysis

Daily to weekly blood count, urinalysis

Blood pressure, monthly cyclosporine level, creatinine/BUN, liver enzymes

Heart rate, blood pressure, creatinine/BUN

Monthly liver enzymes, blood count; consider liver biopsy at 2 gm accumulative dose

Same as by mouth

Heart rate, blood pressure, serum glucose/potassium

Monthly blood count

Weight, blood pressure, serum glucose/potassium, cataract formation

CD19 counts (< 5%), IgG level (keep above 30% of the lower normal limit)

Blood pressure, creatinine/BUN, and electrolytes, monthly trough level (aim 5-15 ng/ml)

Table. Immunosuppressive therapy for inflammatory myopathies

BUN = blood urea nitrogen, GI = gastrointestinal, IgG = immunoglobulin G, IM = intramuscular, IV = intravenous

THERAPY FOR INFLAMMATORY MYOPATHIES

Page 9: NM Update Course II: Inflammatory Myopathies/Myotonic

9

NEUROMUSCULAR UPDATE II

to reach the maximum weekly dose of 25 mg. Daily folic acid 0.8-1.0 mg orally is also administered to prevent stomatitis. Some advocate skipping folic acid on the day of MTX administration.

Besides stomatitis, potential adverse events include alopecia, pneu-monitis, teratogenicity, induction of malignancy, susceptibility to infections, and renal insufficiency. For bone marrow suppression and liver toxicity, complete blood count, differential count, and liver function tests are monitored weekly in the first 4 weeks, then monthly for 6 months, and every 3 months thereafter while on a stable dose. Methotrexate-induced pneumonitis can be difficult to distinguish from myositis-associated interstitial lung disease (ILD). It is not used in patients with known ILD or in those with the Jo-1 antibodies.

Therapeutic effects of oral MTX are often noticeable after 4-8 weeks. If no improvement is observed by that time, the dose is esca-lated. In nonresponders and in more severe cases, the author recom-mends MTX IV or intramuscular treatment at a dose of 0.4-0.8 mg/kg weekly injections, increasing it by 5 mg every week to reach up to 60 mg weekly. Leucovorin rescue on the day after parenteral MTX administration is needed for doses as high as 50 mg.

Azathioprine

Azathioprine, an anti-metabolite that blocks T-lymphocyte prolifera-tion, is a very effective second-line steroid-sparing immunosuppres-sant with delayed onset of response. It is administered in divided doses of 2-3 mg/kg/day, ranging 100-250 mg/day. Start with 50 mg/day for a week before gradually increasing the dose over 1-2 weeks to 100-150 mg/day. In 3-6 months, the author may increase it to the maximum range of 200-250 mg/day. Onset of response is delayed to at least 4-8 months and peaks at 1-2 years. It is therefore not surpris-ing that the 3-month placebo controlled trial of AZA did not show any efficacy.3 However, hand grip strength improvement after 1 year was no different when comparing the AZA to MTX recipients.6 Prior to AZA initiation, it has recently been suggested to test for thiopurine methyltransferase activity. Its deficiency predicts an increased risk of hematologic toxicity. In homozygous cases AZA is contraindi-cated in while in heterozygous patients lower doses may be care-fully tried.15,16 A metanalysis of 54 observational studies and one randomized controlled trial did not demonstrate sufficient evidence to address the effectiveness of thiopurine methyltransferase activity pretesting. In clinical practice, the author monitor spatients’ blood cell counts weekly at the initiation of AZA, then monthly.15 A flu-like reversible acute hypersensitivity reaction affects 12% of users in the first 2 weeks of therapy. It is associated with a rash, elevation in liver enzymes, and pancreatitis. Some may tolerate a re-challenge. Delayed adverse events include myelosuppression, hepatotoxicity, suseptibility to infection, malignancy, teratogenicity, rash, alopecia, fever, and arthralgias.

As long as the patient remains on stable AZA doses, CBC and liver enzymes should be monitored every week for 4 weeks, then monthly for 6 months, and then, for the risk for delayed toxicity, every 3 months. When liver enzymes are markedly elevated (above two times the normal limit), AZA should be stopped for several months until enzymes normalize before the patient may be re-challenged, at times successfully. It is important to obtain levels of liver-specific gamma-glutamyl transferase since transaminases may be released from necrotic muscle fibers.

The dose is adjusted to treatment response and to maintain the white cell count above 3,500 and the absolute lymphocyte count below 1,000. If the white cell count falls below 2,500 or the absolute neutrophil count is below 1,000, AZA administration must be inter-rupted. Patients taking allopurinol, an inhibitor of the main detoxifi-cation pathway, require AZA dose reduction to 25-33% of the above. Angiotensin converting enzyme inhibitors must be avoided due to the serious risk of severe leucopenia.

Intravenous Immunoglobulin

Intravenous immunoglobulin, a pooled gammaglobulin product from several thousand blood donors, has a complex immunomodu-latory mechanism of action. Potential mechanisms include modula-tion and inhibition of pathogenic autoantibodies; suppression of proinflammatory cytokines; blockade of Fc receptor, macrophage colony stimulating factor, and monocyte chemotactant protein-1 in-crease; alteration of T cell function; suppression of circulating CD54 lymphocytes; and inhibition of cell transmigration into the muscle. A randomized-controlled trial with optional cross-over showed that IVIg 2 gm/kg administered monthly for 3 months was very effective (9/12) in treatment-resistant DM.5 Though prospective controlled trials are lacking, IVIg is also believed to be effective in PM17,18 and NM. The American Academy of Neurology 2012 guidelines recommend IVIg as possibly effective and to be considered for treat-ing nonresponsive DM cases.19 There was insufficient evidence to support or refute the use of IVIg in PM or inclusion body myositis. In a recent retrospective inception cohort, all 78 JDM cases were treated steroids but 30 were treated additionally with IVIg.20 The IVIg group maintained similar or lower disease activity than control subjects. To achieve a more rapid improvement in severely affected patients IVIg in instituted in addition to IV solumedrol as initial therapy. Occasionally, the author will administer IVIg as mainte-nance therapy in otherwise refractory patients, or more commonly to reduce long-term CS dose. Dosing is 2 g/kg total initially, given divided over 2-5 days, and then infusions are repeated every 2-4 weeks, with a total monthly dosage of 0.4-2.0 g/kg.

Patients are closely monitored with the first infusion, starting at a very slow rate of 25-50 cc/hr for 30 min, which is increased pro-gressively by 50 cc/hr every 15-20 min up to 150-200 cc/hr. Mild reactions (headache, nausea, chills, myalgia, chest discomfort, and back pain) occur in 10% and are improved with slowing the infusion rate and are preventable with premedication with acetaminophen, benadryl, and if need be IV methylprednisolone. Moderate rare re-actions include chemical meningitis and delayed red, macular skin reaction of the palms, soles, and trunk with desquamation. Acute renal failure is uncommon and related to patient dehydration and the sucrose or maltose diluent. Other severe and rare reactions are anaphylaxis, stroke, myocardial infarction, or pulmonary emboli due to hyperviscocity syndrome. The latter is more likely to occur in old age, immobility, diabetes, thrombocytemia, hypercholesterolemia, hypergammaglobunemia, and cryoglobunemia. Avoid using IVIg in patients with several of these risk factors and place IVIg recipients on low-dose aspirin prophylactically. The extremely rare patients with total IgA deficiency should not be receiving IVIg. In an uncon-trolled preliminary report, seven Caucasian women (four with DM and three with PM) with median disease duration of 72 months were treated with subcutaneous IG.20 Over a median followup period of 14 months, all patients showed a favorable clinical response leading

Page 10: NM Update Course II: Inflammatory Myopathies/Myotonic

10

to reduction of the daily maintenance prednisone dose by a mean of 23 mg and three patients were able to discontinue prednisone altogether.21 These encouraging findings have not been not been validated in a controlled study.

Refractory Patients

In refractory patients, mycophenylate mofetil22 is started as a third-line agent and, in severe cases, rituximab23,24 or cyclophosphamide as well. Other third-line drugs include etanercept,9 cyclosporine, tacro-limus,25 and chlorambucil26 (Table). A large multicenter clinical trial to clarify the role of CD20 depletion using rituximab in refractory PM and DM adults and children was conducted. The results were presented at the 2010 American College of Rheumatology Annual meeting and the article is currently in press.27 Two-hundred patients with DM, JDM, or PM were randomized to receive rituximab early (Group A) or late (8 weeks later, Group B) in the course of this 44-week trial. The primary endpoint measure compared the time to achieve the definition of improvement (DOI) on two consecutive visits between both groups. The secondary endpoint measure was the difference in time to achieve 20% improvement on manual muscle testing on two consecutive visits between both groups. Another secondary endpoint measure was to compare between groups the proportion of patients achieving the DOI at week 8. There were no significant differences between Group A and B in the primary (20.2 and 20.0 weeks, respectively) and in the secondary endpoints since the proportion of early and late rituximab patients achieving DOI at week 8 was 21% and 15%, respectively (p = 0.32). It is likely that the study-delayed treatment design hampered the detection of a signifi-cant benefit of rituximab as 83% of refectory cases met the DOI fol-lowing rituximab treatment. In a small uncontrolled study, rituximab improved six of eight refractory signal recognition particle-positive patients on manual muscle strength and/or resulted in creatine kinase (CK) decline as early as 2 months after treatment.28 Quantitative levels of serum anti-SRP antibodies also decreased after rituximab treatment. Furthermore, anti-SRP-positive myositis appears to be one of the few autoimmune diseases in which specific autoantibody levels are correlated with surrogate disease activity markers.29

In a controlled trial of etanercept, five of 11 subjects in the etaner-cept arm were successfully weaned off prednisone whereas none of the five placebo-recipients could be weaned off.9 The median of the average prednisone dosage after week 24 was lower in the etanercept group (1.2 mg/day) than in the placebo group (29.2 mg/day). Five etanercept-treated subjects and one placebo-treated case developed worsening DM rash. Although a case of refractory DM-ILD was successfully treated with adalimumab,30 another case of rheumatoid arthritis (RA) developed DM 4.5 years after treatment with this anti-tumor necrosis factor (TNF)-α drug.31 The TNF antagonists are widely used for the treatment of rheumatoid arthritis (RA), as well as for inflammatory bowel disease, psoriasis, psoriatic arthritis, and ankylosing spondylitis. Lupus-like symptoms have been reported in < 0.2% of patients within 4-9 months of treatment for these diseases, but development of PM/DM is limited to patients with underly-ing RA. It is likely in the latter case that DM is a manifestation of overlap syndrome rather it being induced by adalimumab. Until an association of PM/DM with TNF antagonists is demonstrated in patients who do not have RA, it is premature to ascertain a causal relationship.

IDIOPATHIC INFLAMMATORY MYOSITIS ASSOCIATED WITH INTERSTITIAL LUNG DISEASE

Corticosteroids are the first-line drug for idiopathic inflammatory myositis (IIM) associated with ILD, but most patients require an adjuvant immunomodulating drug.32 In cases of ILD refractory to steroids, mycophenolate mofetil,33 cyclosporine, and tacrolimus have been shown to be effective second-line agents.25 Early intervention with prednisolone and cyclosporin A (CsA) combination therapy and tight control of the daily CsA dose (by monitoring the blood level 2 hours post-dosing) improved pulmonary function test and chest imaging findings in DM cases with acute-to-subacute ILD.34 Rituximab and cyclophosphamide are third-line options to arrest progression in cases of refractory ILD. A third of treated cases experience resolution of pulmonary involvement, whereas 16% de-teriorated.35 Factors predictive of poor ILD prognosis include older age, symptomatic ILD, lower values of vital capacity and diffusing capacity for carbon monoxide, a pattern of interstitial pneumonia on high resolution computed tomography scan and lung biopsy, and steroid-refractory ILD. There is increased mortality rate in patients with deteriorating ILD as compared to those without ILD deteriora-tion (47.1% versus 3.3%).

PROMISING THERAPIES

Tocilizumab, a humanized anti-IL-6 receptor antibody, is Food and Drug Administration (FDA)-approved for the treatment of moderate-to-severe rheumatoid arthritis.36 Ogata and Tanaka recently reported the first two cases of refractory PM who responded to tocilizumab with reduction in CK as well as in one of the cases resolution of myalgia and stabilization of disease activity and in the other disap-pearance of the high-intensity zones in the thigh muscles on magnetic resonance imaging.36 Abatacept, a fusion protein between Ig and the extracellular domain of cytotoxic T-lymphocyte antigen 4, exerts its anti-inflammatory effect by down-regulating T-cell activation and is FDA-approved for moderate-to-severe rheumatoid arthritis. A refractory JDM case responded to abatacept IV and topical sodium thiosulfate (which promotes vasodilation and vascularization) with significant reductions in muscle and skin inflammation, decreased corticosteroid dependence, and an arrest of calcinosis progression.36 A bedridden PM patient had a very good clinical response to abata-cept.37,38 Therefore, abatacept may hold promise as steroid-sparing agent for the treatment of refractory DM.

There are anecdotal reports of immune ablation similar to that achieved following myeloablative autologous hematopoietic stem cell transplantation through the intensive administration of alem-tuzumab, an anti-CD52 antibody. A patient with refractory PM re-sponded rapidly to a single course of treatment with alemtuzumab.39 Another case of refractory juvenile PM treated with alemtuzumab had stable clinical improvement for more than 6 years.40 However, a refractory adult PM case developed Epstein-Barr virus-driven lym-phoproliferative disorder 9 weeks after alemtuzumab therapy.41 In a preliminary report, 10 patients with refractory or severe DM/PM underwent allogeneic mesenchymal stem cell transplantation and all initially improved in their serum CK, patient global assessment by visual analogue scale, and muscle strength by manual muscle test.42 However, three had recurrence of disease activity 6-8 months after the transplant of whom two underwent a second transplant with good

THERAPY FOR INFLAMMATORY MYOPATHIES

Page 11: NM Update Course II: Inflammatory Myopathies/Myotonic

11

NEUROMUSCULAR UPDATE II

clinical response in one case. The efficacy of allogeneic mesenchy-mal stem cell transplantation in DM or PM has yet to be confirmed in controlled trials.

Recently an old therapy, intramuscular ACTH, has been reported to be effective for inflammatory myopathy.42a This drug is surpris-ingly FDA-approved for the treatment of dermatomyositis and polymyositis.

PHYSICAL THERAPY

In addition to pharmacotherapy, it is generally agreed upon that physical therapy, orthotic devices, occupational therapy, and ex-ercise are beneficial in DM, PM, and NM as early as 2-3 weeks from the acute phase.43 While other studies have reported the safety and benefits of resistive exercise in active patients 1-3 months into their treatment,44,45 most of the studies have been in chronic PM or DM.45 All studies demonstrated the efficacy and safety of exercise as measured by the Functional Index, SF-36, muscle histology, muscle magnetic resonance imaging scanning, or creatine phosphokinase levels. Besides improved muscle strength and increased maximal oxygen uptake, resistance exercise training of eight myositis patients resulted in marked reductions in gene expression, reflecting reduc-tions in proinflammatory and profibrotic gene networks, together with a reduction in tissue fibrosis.46 In severe cases, the author will start with passive range of motion exercises initially and will usually wait for the first month to 3 months for strength and CK to start re-sponding to pharmacotherapy before subjecting severely weakened muscles to a rigorous strengthening exercise program. In patients with mild-to-moderate weakness, the author will initiate a strength-ening program after 2-4 weeks of steroid initiation. Since pain from arthralgia and possibly arthritis is relieved by joint flexion, early mo-bilization is important to prevent flexion contractures of the large and small joints, especially in JDM. There may also be a role for creatine monohydate supplementation as it improves functional performance without significant adverse effects.47

PROGNOSIS

The prognosis of DM, PM, and NM is in general favorable with some exceptions. An associated malignancy portends a poor prognosis for recovery and increases mortality. SANAM is resistant to treatment. Concomitant ILD or Jo-1 or signal recognition particle autoantibod-ies predict a poorer prognosis. Overall, drug-free remissions are rare except in JDM. Recent series underline that only 20-40% of treated patients will achieve PM/DM remission, whereas 60-80% will expe-rience a polycyclic or chronic continuous course of the disease.48,49 On medium- and longterm followup, up to 80% of treated PM/DM patients are still disabled based on Health Assessment Questionnaire scores.50 The overall mortality ratio in PM/DM patients also remains two- to threefold higher compared with the general population, with cancer, lung, and cardiac complications and infections being the most common causes of death.51,52 Poor prognostic factors in PM/DM patients include older age,53 male gender, non-Caucasian ethnicity, longer symptoms duration, ILD,54 cardiac involvement, dysphagia,55 cancer,52 and serum myositis-specific antibodies (in-cluding coexistence of anti-Ro52 and anti-Jo1 antibodies, presence of anti-signal recognition particle antibody, anti-155/140, and anti-

CADM-140 antibodies). Complete remission of PM/DM was less frequent (13.6% versus 41.1%) and the mortality rate (47.8% versus 7.3%) was higher in elderly patients than in younger patients.53 In a recent series, the coexistence of Ro52 and Jo1 antibodies was associ-ated with more severe myositis/joint impairment, symptomatic ILD, increased risk of cancer, and higher mortality.56 Anti-SRP antibody is associated with acute onset of refractory necrotizing myositis and antibody titers correlate with CK levels and disease activity.57 Anti-155/140 antibody is associated with malignancy, whereas the pres-ence of anti–CADM-140 antibody is associated with amyopathic DM and rapidly progressive ILD.

Page 12: NM Update Course II: Inflammatory Myopathies/Myotonic

1212

REFERENCES

1. Amato AA, Barohn RJ. Evaluation and treatment of inflammatory myopathies. J Neurol Neurosurg Psych 2009;80:1060-1068.

2. Grable-Esposito P, Katzberg HD, Greenberg SA, Srinivasan J, Katz J, Amato AA. Immune-mediated necrotizing myopathy associated with statins. Muscle Nerve 2010;41(2):185-190.

3. Bunch TW, Worthington JW, Combs JJ, Ilstrup MS, Engel AG. Azathioprine with prednisone for polymyositis. Ann Intern Med 1980;92(3):365-369.

4. Miller FW, Leitman SF, Cronin M, et al. Controlled trial of plasma exchange and leukapheresis in polymyositis and dermatomyositis. N Engl J Med 1992;326:1380-1384.

5. Dalakas MC, Illa I, Dambrosia JM, et al. A controlled trial of high-dose intravenous immune globulin infusions as treatment for dermatomyosi-tis. N Engl J Med 1993;329(27):1993-2000.

6. Miller J, Walsh Y, Saminaden S, Lecky BRF, Winer JB. Randomized double blind controlled trial of methotrexate and steroids compared with azathioprine and steroids in the treatment of idiopathic inflamma-tory myopathy. J Neurol Sci 2002;199(Suppl 1):S53.

7. Vencovsky J, Jarosova K, Machacek S, et al. Cyclosporine A versus methotrexate in the treatment of polymyositis and dermatomyositis. Scand J Rheum 2000;29(2):95-102.

8. Villalba L, Hicks JE, Adams EM, et al. Treatment of refractory myosi-tis: a randomized crossover study of two new cytotoxic regimens. Arth Rheum 1998;41(3):392-399.

9. Muscle Study Group. A randomized, pilot trial of etanercept in derma-tomyositis. Ann Neurol 2011;70(3):427-436.

10. van de Vlekkert J, Hoogendijk JE, de Haan RJ, et al. Oral dexa-methasone pulse therapy versus daily prednisolone in sub-acute onset myositis, a randomised clinical trial. Neuromuscul Disord 2010;20(6):382-389.

11. Rodd C, Lang B, Ramsay T, et al. Incident vertebral fractures among children with rheumatic disorders 12 months after glucocorticoid ini-tiation: a national observational study. Arthritis Care Res (Hoboken) 2012;64(1):122-131.

12. Saad CG, Borba EF, Aikawa NE, et al. Immunogenicity and safety of the 2009 non-adjuvanted influenza A/H1N1 vaccine in a large cohort of autoimmune rheumatic diseases. Ann Rheum Dis 2011;70(6):1068-1073.

13. Marie I, Ménard JF, Hachulla E, et al. Infectious complications in poly-myositis and dermatomyositis: a series of 279 patients. Semin Arthritis Rheum 2011;41(1):48-60.

14. Efthimiou P, Pokharna H, Kukar M, Hennessey K. PCP chemoprophy-laxis is essential for lymphopenic dermatomyositis patients treated with immunomodulators. Muscle Nerve 2011;43(6):918-919.

15. Evans WE, Hon YY, Bomgaars L, et al. Preponderance of thio-purine S-methyltransferase deficiency and heterozygosity among patients intolerant to mercaptopurine or azathioprine. J Clin Oncol 2001;19(8):2293-2301.

16. Booth RA, Ansari MT, Loit E, et al. Assessment of thiopurine S-methyltransferase activity in patients prescribed thiopurines: a sys-tematic review. Ann Intern Med 2011;154(12):814-823.

17. Cherin P, Piette JC, Wechsler B, et al. Intravenous gamma globulin as first line therapy in polymyositis and dermatomyositis: an open study in 11 adult patients. J Rheum 1994;21:1092-1097.

18. Danieli MG, Malcangi G, Palmieri C, et al. Cyclosporin A and intrave-nous immunoglobulin treatment in polymyositis/dermatomyositis. Ann Rheum Dis 2002;61:37-41.

19. Patwa HS, Chaudhry V, Katzberg H, Rae-Grant AD, So YT. Evidence-based guideline: Intravenous immunoglobulin in the treatment of neuromuscular disorders: report of the Therapeutics and Technology Assessment Subcommittee of the American Academy of Neurology. Neurology 2012;27;78(13):1009-1015.

20. Lam CG, Manlhiot C, Pullenayegum EM, Feldman BM. Efficacy of intravenous Ig therapy in juvenile dermatomyositis. Ann Rheum Dis 2011;70(12):2089-2094.

21. Danieli MG, Pettinari L, Moretti R, Logullo F, Gabrielli A. Subcutaneous immunoglobulin in polymyositis and dermatomyositis: a novel application. Autoimmun Rev 2011;10(3):144-149.

22. Rowin J, Amato AA, Deisher N, Cursi, J, Meriggioli MN. Mycophenolate mofetil in dermatomyositis: is it safe? Neurology 2006;66(8), 1245-1247.

23. Levine TD. Rituximab in the treatment of dermatomyositis: an open-label pilot study. —Arthritis Rheum 2005;52(2):601-607.

24. Noss EH, Hausner-Sypek DL, Weinblatt ME. Rituximab as therapy for refractory polymyositis and dermatomyositis. J Rheumatol 2006;33(5):1021-1026.

25. Oddis CV, Sciurba FC, Elmagd KA, Starzl TE. Tacrolimus in refractory polymyositis with interstitial lung disease. Lancet 1999;22;353(9166):1762-1763.

26. Cagnoli M, Marchesoni A, Tosi S. Combined steroid, methotrexate and chlorambucil therapy for steroid-resistant dermatomyositis. Clin Exp Rheumatol 1991;9(6):658-659.

27. Oddis CV, Reed AM, Aggarwal R, et al. The RIM study group: Rituximab in the treatment of refractory adult and juvenile dermato-myositis (DM) and adult polymyositis (PM)—The RIM study. Arthritis Rheum 2010:62:3844.

28. Valiyil R, Casciola-Rosen L, Hong G, Mammen A, Christopher-Stine L. Rituximab therapy for myopathy associated with anti-signal recog-nition particle antibodies: a case series. Arthritis Care Res (Hoboken) 2010;62(9):1328-1334

29. Benveniste O, Drouot L, Jouen F, et al. Correlation of anti-signal recog-nition particle autoantibody levels with creatine kinase activity in patients with necrotizing myopathy. Arthritis Rheum 2011;63(7):1961-1971.

30. Park JK, Yoo HG, Ahn DS, Jeon HS, Yoo WH. Successful treatment for conventional treatment-resistant dermatomyositis-associated in-terstitial lung disease with adalimumab. Rheumatol Int 2011 Nov 17. Epub ahead of print.

31. Brunasso AM, Scocco GL, Massone C. Dermatomyositis during adalim-umab therapy for rheumatoid arthritis. J Rheumatol 2010;37(7):1549-1550.

32. Mimori T, Nakashima R, Hosono Y. Interstitial Lung Disease in Myositis: Clinical Subsets, Biomarkers, and Treatment. Curr Rheumatol Rep 2012 Feb 25. Epub ahead of print.

33. Morganroth PA, Kreider ME, Werth VP. Mycophenolate mofetil for interstitial lung disease in dermatomyositis. Arthritis Care Res (Hoboken) 2010;62(10):1496-1501.

34. Kotani T, Takeuchi T, Makino S, et al. Combination with corticoste-roids and cyclosporin-A improves pulmonary function test results and chest HRCT findings in dermatomyositis patients with acute/subacute interstitial pneumonia. Clin Rheumatol 2011;30(8):1021-1028.

35. Marie I, Hatron PY, Dominique S, Cherin P, Mouthon L, Menard JF. Short-term and long-term outcomes of interstitial lung disease in poly-myositis and dermatomyositis: a series of 107 patients. Arthritis Rheum 2011;63(11):3439-3447.

36. Ogata A, Tanaka T. Tocilizumab for the treatment of rheumatoid arthri-tis and other systemic autoimmune diseases: current perspectives and future directions. Int J Rheumatol 2012;2012:946048.

37. Arabshahi B, Silverman RA, Jones OY, Rider LG. Abatacept and sodium thiosulfate for treatment of recalcitrant juvenile dermatomyositis complicated by ulceration and calcinosis. J Pediatr 2012;160(3):520-522.

38. Musuruana JL, Cavallasca JA. Abatacept for treatment of refractory polymyositis. Joint Bone Spine 2011;78(4):431-432.

39. Thompson B, Corris P, Miller JA, Cooper RG, Halsey JP, Isaacs JD. Alemtuzumab (Campath-1H) for treatment of refractory polymyositis. J Rheumatol 2008;Oct;35(10):2080-2082.

THERAPY FOR INFLAMMATORY MYOPATHIES

Page 13: NM Update Course II: Inflammatory Myopathies/Myotonic

13

NEUROMUSCULAR UPDATE II

40. Reiff A, Shaham B, Weinberg KI, Crooks GM, Parkman R. Anti-CD52 antibody-mediated immune ablation with autologous immune recovery for the treatment of refractory juvenile polymyositis. J Clin Immunol 2011;31(4):615-622.

41. Cooles FA, Jackson GH, Menon G, Isaacs JD. Epstein-Barr virus-driven lymphoproliferative disorder post-CAMPATH-1H (alemtuzumab) in refractory polymyositis. Rheumatology (Oxford) 2011;50(4):810-812.

42. Wang D, Zhang H, Cao M, et al. Efficacy of allogeneic mesenchymal stem cell transplantation in patients with drug-resistant polymyositis and dermatomyositis. Ann Rheum Dis 2011;70(7):1285-1288.

42a. Levine T. Treating refractory dermatomyositis or polymyositis with adrenocorticotropic hormone gel: a retrospective case series. Drug Des Devel Ther 2012;6:133-139.

43. Varjú C, Pethö E, Kutas R, Czirják L. The effect of physical exercise following acute disease exacerbation in patients with dermato/poly-myositis. Clin Rehabil 2003;17(1):83-87.

44. Alexanderson H, Stenström CH, Jenner G, Lundberg I. The safety of a resistive home exercise program in patients with recent onset active polymyositis or dermatomyositis. Scand J Rheumatol 2000;29(5):295-301.

45. Alexanderson H. Exercise effects in patients with adult idiopathic inflammatory myopathies. Curr Opin Rheumatol 2009;21(2):158-163.

46. Nader GA, Dastmalchi M, Alexanderson H, et al. A longitudinal, in-tegrated, clinical, histological and mRNA profiling study of resistance exercise in myositis. Mol Med 2010;16(11-12):455-464.

47. Chung YL, Alexanderson H, Pipitone N, et al. Creatine supplements in patients with idiopathic inflammatory myopathies who are clini-cally weak after conventional pharmacologic treatment: Six-month, double-blind, randomized, placebo-controlled trial. Arthritis Rheum 2007;57(4):694-702.

48. Marie I, Hachulla E, Hatron PY, et al. Polymyositis and dermato-myositis: short term and longterm outcome, and predictive factors. J Rheumatol 2001;28:2230-2237.

49. Bronner IM, van der Meulen MF, de Visser M, et al. Long-term outcome in polymyositis and dermatomyositis. Ann Rheum Dis 2006;65:1456-1461.

50. Ponyi A, Borgulya G, Constantin T, et al. Functional outcome and quality of life in adult patients with idiopathic inflammatory myositis. Rheumatology (Oxford) 2005;44:83-88.

51. Limaye V, Hakendorf P, Woodman RJ, Blumbergs P, Roberts-Thomson P. Mortality and its predominant causes in a large cohort of pa-tients with biopsy-determined inflammatory myositis. Intern Med J 2012;42(2):191-198.

52. Airio A, Kautiainen H, Hakala M. Prognosis and mortality of polymyo-sitis and dermatomyositis patients. Clin Rheumatol 2006;25:234-239.

53. Marie I, Hatron PY, Levesque H, et al. Influence of age on charac-teristics of polymyositis and dermatomyositis in adults. Medicine (Baltimore) 1999;78:139-147.

54. Yamasaki Y, Yamada H, Ohkubo M, et al. Longterm survival and asso-ciated risk factors in patients with adult-onset idiopathic inflammatory myopathies and amyopathic dermatomyositis: experience in a single institute in Japan. J Rheumatol 2011;38:1636-1643.

55. Danko K, Ponyi A, Constantin T, et al. Long-term survival of pa-tients with idiopathic inflammatory myopathies according to clinical features: a longitudinal study of 162 cases. Medicine (Baltimore) 2004;83:35-42.

56. Marie I, Hatron PY, Dominique S, et al. Short-Term and Long-Term Outcome of Anti-Jo1-Positive Patients with Anti-Ro52 Antibody. Semin Arthritis Rheum 2011 Nov 9. Epub ahead of print.

57. Benveniste O, Drouot L, Jouen F, et al. Correlation of anti-signal rec-ognition particle autoantibody levels with creatine kinase activity in pa-tients with necrotizing myopathy. Arthritis Rheum 2011;63:1961-1971.

Page 14: NM Update Course II: Inflammatory Myopathies/Myotonic

14

Page 15: NM Update Course II: Inflammatory Myopathies/Myotonic

15

NEUROMUSCULAR UPDATE II

15

Inflammatory Myopathies

Steven A. Greenberg, MDAssociate Professor of Neurology

Department of Neurology, Division of Neuromuscular DiseaseBrigham and Women’s Hospital

andChildren’s Hospital Informatics Program

Harvard Medical SchoolBoston, Massachusetts

CASE PRESENTATION

Case One

A 35-year-old woman developed difficulty arising from a chair, climbing stairs, washing her hair, and removing items from shelves in her closet. Symptoms progressed in intensity for 3 months and she then developed a fine pruritic rash along her upper anterior chest and dry, cracking hands with discoloration on the back of her fingers. Physical examination showed an ery-thematous rash around the eyes and on the anterior chest, purplish discoloration of papules on the back of the hands, particularly over the joints, and moderate weakness of neck flexion, shoulder abduction, arm flexion, hip flexion, and knee flexion. She was unable to arise from a chair without using her hands to push off. Laboratory studies showed a normal serum creatine kinase (CK) (63 IU/ml). Complete blood count, electrolytes, antinuclear antibodies (ANA), and aspartate aminotransferase/alanine amino-transferase were normal. The muscle biopsy is shown in Figure 1.

Figure 1. Hematoxylin and eosin stained sections from muscle biopsy Case One. Used with permission from the author.

Case Two

A 67-year-old man developed slowly progressive difficulty walking with a tendency for his knees to buckle, difficulty climb-ing stairs, and difficulty holding eating utensils and turning keys. Serum CK was 845 IU/L. He underwent muscle biopsy, was diagnosed with polymyositis (PM), and treated with prednisone 60 mg orally daily. Three months later followup CK was 252 IU/L but there was no improvement in strength. He continued to weaken, while remaining on high-dose prednisone, which was eventually tapered 1 year later. He underwent another consulta-tion, which showed generalized weakness with a marked prefer-ence for wrist and finger flexors (Figure 2), and quadriceps, with relatively spared arm abductors. He underwent repeat muscle biopsy (Figure 3).

Figure 2. Preferential involvement of finger flexors over finger extensors in Case Two. Used with permission from the author.

Page 16: NM Update Course II: Inflammatory Myopathies/Myotonic

16

Figure 3. Muscle pathology of second biopsy, hematoxylin and eosin staining, Case Two. Inset shows enlargement of the central myofiber. Used with permission from the author.

CASE DISCUSSIONS

Case One

This patient developed proximal weakness along with a skin rash, immediately raising the possibility of dermatomyositis (DM). The description of the rash including purplish papules over the dorsal finger joints suggests the presence of Gottron’s papules. The normal serum CK makes unlikely certain inherited muscle diseases but does not diminish the possibility of DM, as CKs are often normal in DM. The muscle biopsy demonstrates perivas-cular and perimysial location of inflammatory cells (Figure 1A) and the presence of perifascicular atrophy (Figures 1A and 1B) that is evident as basophilic atrophic fibers around the periphery of fascicles. Perifascicular atrophy is virtually diagnostic of DM.

Case Two

This patient developed slowly progressive weakness of the hands and quadriceps by history and by physical examination had marked weakness of finger flexors in contrast to finger extensors. This pattern immediately suggests sporadic inclusion body myo-sitis (sIBM). The muscle biopsy (Figure 3) shows endomysial inflammation with invasion of non-necrotic muscle fibers (arrow) and rimmed vacuoles (arrowheads). The lack of clinical response to prednisone, despite reduction in serum CK values, is charac-teristic, as is the initial diagnosis as PM.

GENERAL DISCUSSION

The inflammatory myopathies are diseases in which muscle appears to be injured by the immune system. The clinical features and pathogenesis have been recently reviewed and much of the material that follows is adapted from such recent reviews.1-5

The principal subtypes are DM, inclusion body myositis (IBM), and PM, though many patients have syndromes that are not easily classified and may best be labeled as nonspecific (or unspecified) myositis. Other subtypes include necrotizing myopathy, overlap syndromes (inflammatory myopathy occurring in a patient with

a connective tissue disorder such as mixed connective tissue disease), granulomatous myositis, and eosinophilic myositis. The mechanisms initiating and maintaining these diseases are not well understood.6

Dermatomyositis

Dermatomyositis affects children and adults. Adult DM generally presents as subacute progressive painless proximal weakness, a skin rash, or both. Juvenile DM (JDM) may present similarly or as an acute or subacute febrile illness followed by skin, muscle, or sometimes multisystem involvement. The mechanism of disease is associated with induction of the type I interferon pathway.4

The skin involvement in DM may have diverse manifesta-tions, including a heliotrope rash (purplish discoloration) on the eyelids; an erythematosus rash on the face, neck, and anterior chest (“V-sign”), upper back (“shawl sign”), elbows, or knees; a purplish scaly papular rash on the extensor surface of the hands (Gottron’s papules); thickened and cracked skin on the dorsal and ventral surfaces of the hands (“mechanic’s hands”); and other changes. Subcutaneous calcinosis is a significant problem in JDM and uncommon in adult DM. Cutaneous symptoms in DM have a high impact on lowering quality of life in patients and include prominent pruritus.7,8

The pattern of proximal limb weakness in DM is not distinc-tive and does not distinguish DM from many other myopathies. Significant muscle asymmetries or prominent distal (forearm or lower leg) weakness together with skin rash should prompt con-sideration for sarcoidosis, for which clinical involvement similar to DM has been recognized.9 Normal serum CK may be present in patients with progressing disease and does not exclude the diagnosis. When elevated serum CK is present in DM, reductions generally occur with treatment, and elevation with relapse.

Additional evaluation of adult patients with DM should be performed because of its association with two other important clinical syndromes: interstitial lung disease and malignancy. Pulmonary function tests, chest computed tomography (CT), and laboratory testing for the presence of antihistidyl transfer RNA antibodies (anti-Jo-1 antibodies) should be considered in all patients with DM. Malignancy has been estimated to be associ-ated with 6-45% of adult patients with DM, with age-associated increased risk particularly in women over 40 years old. A malig-nancy evaluation—including physical examination (skin exami-nation, breast and pelvic examinations in women, and testicular and prostate examination in men), blood studies (complete blood count, liver function tests, lactate dehydrogenase, and prostate specific antigen), stool studies for occult blood, CT (chest, abdomen, and pelvis), and colonoscopy—should be considered in every adult patient with a new diagnosis of DM.

Muscle biopsy is an important diagnostic procedure in DM. The clinical syndrome in patients with typical skin and muscle features is quite specific for DM though some patients with sarcoidosis have been reported with similar clinical but distinct pathological features.10-12 The most supportive diagnostic feature of muscle pathology for DM evident in routine clinical studies is the presence of perifascicular atrophy, small myofibers that are

INFLAMMATORY MYOPATHIES

Page 17: NM Update Course II: Inflammatory Myopathies/Myotonic

17

PRINCIPLES OF NCS AND NEEDLE EMG

slightly darker and bluish in color in hematoxylin and eosin sec-tions, typically located at the edges of fascicles.

Inclusion Body Myositis

Inclusion body myositis affects adults during middle age and later life. The name was first applied to a patient with symptom onset at age 18 and findings at age 26 consisting of lordotic posture, leg limb-girdle weakness, and no atrophy or weakness of the quadriceps. This patient would not meet current criteria for the diagnosis of IBM.13 Although onset over age 50 has been em-phasized, symptom onset prior to age 50 is common (18-20% of patients).14,15 Diagnosis has historically been frequently delayed by a mean of 5-8 years from symptom onset.14,16-19 The mecha-nism of disease may be linked to nuclear degeneration and the redistribution of the normally nuclear protein TDP-43 to the sarcoplasm.20,21

The clinical presentation of IBM is quite distinct from that of other inflammatory myopathies. Atrophy and weakness of wrist and finger flexors and quadriceps are distinctive and physical examination should focus on careful testing of these muscle groups. Comparison of wrist and finger extensors with corre-sponding flexors may demonstrate the greater involvement of the flexors and asymmetries. Relative preservation of deltoids, in comparison to the forearm flexors, can be impressive, in marked contrast to the pattern of weakness seen in DM and PM. Contrasts between severe biceps weakness, but better preserved brachioradialis, and severe deep finger flexor weakness, but un-commonly involved adductor pollicis, have been emphasized as well.15 Involvement of tibialis anterior may also be distinctive in IBM. Dysphagia can be a significant problem, with a prevalence estimated as high as 66%.15

Serum CK is only modestly elevated; research criteria have pro-posed diagnostic criteria of an upper limit of 12 times the upper limit of normal,22 though patients with higher values, up to 16 times the upper limit of normal, have been reported.15 Serum electrophoresis and the more sensitive immunofixation should be considered because some patients have a detectable serum mono-clonal immunoglobulin population.

The presence of multiple myofibers surrounded by inflamma-tory cells and many myofibers with rimmed vacuoles is highly supportive of a pathological diagnosis of IBM. Both IBM and PM (see below) may have similar patterns with respect to the location of inflammatory cells as seen in routine studies. The pattern of inflammatory cells deep within fascicles surrounding and sometimes invading myofibers is distinct from that of DM. What distinguishes IBM from PM in light microscopic examina-tion is a sufficient number of rimmed vacuoles, though diagnostic and research criteria for what constitutes sufficient numbers of rimmed vacuoles have not been established. The presence of cy-tomembranous whorls and filamentous inclusions with electron microscopy are also highly supportive of a diagnosis of IBM. Difficulties with diagnosis occur in patients with typical clinical features but few inflammatory cells or with few rimmed vacuoles. Small numbers of rimmed vacuoles may be seen in patients with steroid-responsive PM syndromes.23

Polymyositis

Patients with acquired myopathies whose weakness improves with immunosuppressive therapies and relapses with taper of such therapy, but lack the rash and pathological features of DM are challenging to classify. Depending on various criteria, such patients may be categorized as having PM, nonspecific myositis, necrotizing myopathy, overlap syndromes, or other diagnoses.

Patients with subacute progressive symmetrical proximal arm and leg weakness, without skin rash, and with muscle biopsy features of prominent inflammatory cells surrounding many muscle fibers, without perifascicular atrophy, are the patients that are most appropriately classified as having PM or nonspecific myositis. Various research diagnostic criteria have been considered with regard to the challenges of PM diagnosis.24-26 The practical issues are avoiding misclassification of certain muscular dystrophies, particularly limb-girdle muscular dystrophy (LGMD), and IBM as PM. Most patients with LGMD and IBM meet widely-used criteria for the diagnosis of PM.27

As with DM, there is an association with interstitial lung disease, but not a well-established association with malignancy. Serum CK is almost always elevated in patients with progressing PM. Connective tissue diseases should be considered through clinical evaluation and antinuclear antibody testing.

The pathological diagnosis of PM is reasonable when there are abundant inflammatory cells surrounding multiple myofibers and an absence of rimmed vacuoles. There are considerable differ-ences in opinion, ranging from support for diagnostic criteria that allow a diagnosis of definite PM with biopsy features that include some unspecified combination of muscle degeneration, regenera-tion, necrosis, and inflammatory cells,28 to positions that the di-agnosis should require invasion by CD8+ T cells (which include cytotoxic and suppressor T cells) of non-necrotic muscle fibers with visible expression of major histocompatibility class 1 by immunohistochemistry on these fibers’ sarcolemma. The former criteria may lead to misdiagnosis of genetically determined myopathies, which may have variable degrees of inflammation present (the association of calpain mutations with eosinophilic myositis is one excellent example of this problem29), and IBM as PM, while the latter is restrictive enough that many patients with immune-mediated myopathies need an alternative diagnosis, such as non-specific or unspecified myositis.26

Pathogenesis of Dermatomyositis

The pathophysiology of DM appears to be associated with the family of type I IFN cytokines.4,6 Microarray studies of muscle biopsy specimens generating over 2 million data points assay-ing gene expression of tens of thousands of genes in more than 100 muscle biopsy samples has found the marked overproduc-tion of type I interferon (IFN)-inducible transcripts and proteins in muscle to be remarkably unique to DM in comparison to all other muscle diseases studied.4 Elevation of type I IFN-inducible transcripts has remarkable specificity for DM. For example, the transcript for the type I IFN-stimulated gene 15 (ISG15) was higher in muscle in all 28 biopsies from adults with DM and

Page 18: NM Update Course II: Inflammatory Myopathies/Myotonic

18

perifascicular atrophy and children with JDM than in every one of 199 non-DM biopsy samples from a wide range of neuromus-cular diseases.

Some type I IFN-inducible proteins also are highly specific bio-markers of DM muscle. MxA, for example, is impressively and uniquely (in comparison to other muscle diseases) abundant in DM myofibers with perifascicular atrophy and in DM capillaries. An ubiquitin-like modifier, ISG15 is conjugated to many other proteins in DM muscle. In vitro models of human skeletal muscle cell cultures, exposed to IFN-α or IFN-β, produces a similar picture of ISG15 conjugation present in human DM samples.

Which specific type I IFN might be driving the marked type I IFN-inducible response in DM muscle is uncertain but recent evidence suggests that it is interferon-®.30

Pathogenesis of Inclusion Body Myositis

There are many theories of the pathogenesis of IBM. Toxicity theo-ries of various molecules have dominated the field. Mitochondrial pathology and infectious causes have also been considered. Other considerations have included a potential infectious origin and a mitochondrial disorder. Autoimmunity and nuclear degeneration are perhaps the strongest theories.

Autoimmunity

The adaptive immune system is an arm of the immune system capable of generating highly specific molecular targeting. Inclusion body myositis has a highly refined adaptive immune response. Both T cells and B cells undergo affinity maturation, selection, and clonal expansion through high-affinity interactions of their receptors (the T cell antigen receptor and B cell surface immunoglobulin receptor) with target antigen.

Cytotoxic T cell invasion of myofibers has been emphasized since the mid-1980s. Molecular analyses of the T cell receptor (analysis of the nucleotide sequences of the variable regions of their α and β chains) have demonstrated clonal restriction (a limited population of distinct T cell receptor TCR transcript se-quences), providing circumstantial evidence of clonal expansion (that T cells have been activated by antigen and proliferated). In IBM muscle, antigen-stimulated T cells develop highly specific antigen-directed receptors during poliferation.

Inclusion body myositis also has a highly refined B cell response with plasma cells31 with immunoglobulin gene rearrangements, characteristic of clonal expansion in response to local antigen stimulation,32 and a permissive environment for ectopic lymphoid structures suggestive of local maturation of B cells in muscle.33 The differentiated B cells develop into clonally expanded, highly-refined antigen directed plasma cells that produce and secrete immunoglobulins within IBM muscle.

Recognition that a B cell-specific response was present in IBM muscle and characterization of this response suggested that a search for circulating IBM autoantibodies might be fruitful. In 2011, an autoantibody to a human muscle protein of approxi-mately 43 kDa was identified in 52% (13/25) of IBM samples,

0% (0/25) of other autoimmune myopathy samples, and 0% (0/15) of normal samples.34 The identity of this protein has not been established.

Nuclear Degeneration

Rimmed vacuoles in IBM muscle derive from nuclei. They are lined with the nuclear membrane proteins lamin A/C and emerin.21 Lining of these vacuoles with histone H1 and with emerin was reported in 200835 and with histone 2AX and DNA repair regulatory components (DNA-PK, Hu70, and Hu80) in 2011.36

Inclusion body myositis muscle is also notable for the presence of an aberrantly-localized sacroplasmic protein with the capac-ity to nonspecifically bind nucleic acids. The nucleic acid-binding protein TDP-43 has been identified in IBM non-nuclear sarco-plasm.20,37 TDP-43 is a predominantly nuclear heterogeneous nuclear ribonucleoprotein that undergoes nucleocytoplasmic shut-tling and associates with translation machinery in the cytoplasm.

REFERENCES

1. Greenberg SA. Theories of the pathogenesis of inclusion body myosi-tis. Curr Rheumatol Rep 2010;12:221-228.

2. Greenberg SA. Inclusion body myositis. Curr Opin Rheumatol 2011;23:574-578.

3. Greenberg SA. Pathogenesis and therapy of inclusion body myositis. Curr Opin Neurol 2012; in press.

4. Greenberg SA. Type 1 interferons and myositis. Arth Res Ther 2010;12(Suppl 1):S4.

5. Greenberg SA. American Association of Neuromuscular and Electrodiagnostic Medicine Course: Inflammatory myopathies. Rochester, MN: American Association of Neuromuscular and Electrodiagnostic Medicine; 2010.

6. Greenberg SA. Proposed models of the inflammatory myopathies and their potential therapeutic implications. Neurology 2007;69:2008-2019.

7. Shirani Z, Kucenic MJ, Carroll CL, et al. Pruritus in adult dermatomyo-sitis. Clin Exp Dermatol 2004;29:273-276.

8. Hundley JL, Carroll CL, Lang W, et al. Cutaneous symptoms of der-matomyositis significantly impact patients’ quality of life. J Am Acad Dermatol 2006;54:217-220.

9. Kubis N, Woimant F, Polivka M, et al. A case of dermatomyositis and muscle sarcoidosis in a Caucasian patient. J Neurol 1998;245:50-52.

10. von Wussow P, Jakschies D, Hartung K, Deicher H. Presence of inter-feron and anti-interferon in patients with systemic lupus erythemato-sus. Rheumatol Int 1988;8:225-230.

11. Lipton JH, McLeod BD, Brownell AK. Dermatomyositis and granu-lomatous myopathy associated with sarcoidosis. Can J Neurol Sci 1988;15:426-429.

12. Brateanu AC, Caracioni A, Smith HR. Sarcoidosis and dermatomyosi-tis in a patient with hemoglobin SC. A case report and literature review. Sarcoidosis Vasc Diffuse Lung Dis 2000;17:190-193.

13. Yunis EJ, Samaha FJ. Inclusion body myositis. Lab Invest 1971;25:240-248.

14. Lotz BP, Engel AG, Nishino H, et al. Inclusion body myositis. Observations in 40 patients. Brain 1989;112(Pt 3):727-747.

15. Badrising UA, Maat-Schieman ML, van Houwelingen JC, et al. Inclusion body myositis clinical features and clinical course of the disease in 64 patients. J Neurol 2005;252(12):1448-1454.

16. Badrising UA, Maat-Schieman M, van Duinen SG, et al. Epidemiology of inclusion body myositis in the Netherlands: a nationwide study. Neurology 2000;55:1385-1387.

INFLAMMATORY MYOPATHIES

Page 19: NM Update Course II: Inflammatory Myopathies/Myotonic

19

NEUROMUSCULAR UPDATE II

17. Beyenburg S, Zierz S, Jerusalem F. Inclusion body myositis: clin-ical and histopathological features of 36 patients. Clin Investig 1993;71:351-361.

18. Lindberg C, Persson LI, Bjorkander J, Oldfors A. Inclusion body myo-sitis: clinical, morphological, physiological and laboratory findings in 18 cases. Acta Neurol Scand 1994;89:123-131.

19. Sayers ME, Chou SM, Calabrese LH. Inclusion body myositis: analy-sis of 32 cases. J Rheumatol 1992;19:1385-1389.

20. Salajegheh M, Pinkus JL, Taylor JP, et al. Sarcoplasmic redistribu-tion of nuclear TDP-43 in inclusion body myositis. Muscle Nerve. 2009;40:19-31

21. Greenberg SA, Pinkus JL, Amato AA. Nuclear membrane proteins are present within rimmed vacuoles in inclusion-body myositis. Muscle Nerve 2006;34:406-416.

22. Griggs RC, Askanas V, DiMauro S, et al. Inclusion body myositis and myopathies. Ann Neurol 1995;38:705-713.

23. van der Meulen MF, Hoogendijk JE, Moons KG, et al. Rimmed vacu-oles and the added value of SMI-31 staining in diagnosing sporadic inclusion body myositis. Neuromuscul Disord 2001;11:447-451.

24. Hoogendijk JE, Amato AA, Lecky BR, et al. 119th European Neuro Muscular Centre international workshop: trial design in adult idio-pathic inflammatory myopathies, with the exception of inclusion body myositis, 10-12 October 2003, Naarden, The Netherlands. Neuromuscul Disord 2004;14:337-345.

25. Amato AA, Griggs RC. Unicorns, dragons, polymyositis, and other mythological beasts. Neurology 2003;61:288-289.

26. van der Meulen MF, Bronner IM, Hoogendijk JE, et al. Polymyositis: an overdiagnosed entity. Neurology 2003;61:316-321.

27. Bohan A, Peter JB. Polymyositis and dermatomyositis (first of two parts). N Engl J Med 1975;292:344-347.

28. Kamme F, Salunga R, Yu J, et al. Single-cell microarray analysis in hippocampus CA1: demonstration and validation of cellular heteroge-neity. J Neurosci 2003;23:3607-3615.

29. Krahn M, Lopez de Munain A, Streichenberger N, et al. CAPN3 mu-tations in patients with idiopathic eosinophilic myositis. Ann Neurol 2006;59:905-911.

30. Liao AP, Salajegheh M, Nazareno R, et al. Interferon beta is associated with type 1 interferon-inducible gene expression in dermatomyositis. Ann Rheum Dis 2011;70:831-836.

31. Greenberg SA, Bradshaw EM, Pinkus JL, et al. Plasma cells in muscle in inclusion body myositis and polymyositis. Neurology 2005;65:1782-1787.

32. Bradshaw EM, Orihuela A, McArdel SL et al. A local antigen-driven humoral response is present in the inflammatory myopathies. J Immunol. 2007;178:547-556

33. Salajegheh M, Pinkus JL, Amato AA, et al. Permissive environment for B-cell maturation in myositis muscle in the absence of B-cell follicles. Muscle Nerve 2010;42:576-583.

34. Salajegheh M, Lam T, Greenberg SA. Autoantibodies against a 43 KDa muscle protein in inclusion body myositis. PLoS One. 2011;6:e20266.

35. Nakano S, Shinde A, Fujita K, et al. Histone H1 is released from myo-nuclei and present in rimmed vacuoles with DNA in inclusion body myositis. Neuromuscul Disord 2008;18:27-33.

36. Nishii M, Nakano S, Nakamura S, et al. Myonuclear breakdown in sporadic inclusion body myositis is accompanied by DNA double strand breaks. Neuromuscul Disord 2011;21:345-352.

37. Weihl CC, Temiz P, Miller SE, et al. TDP-43 accumulation in inclu-sion body myopathy muscle suggests a common pathogenic mecha-nism with frontotemporal dementia. J Neurol Neurosurg Psychiatry 2008;79:1186-1189.

Page 20: NM Update Course II: Inflammatory Myopathies/Myotonic

20

Page 21: NM Update Course II: Inflammatory Myopathies/Myotonic

21

Myotonic Dystrophies

Bassam A. Bassam, MDProfessor of Neurology

Director, Neuromuscular and EMG LaboratoryUniversity of South Alabama

Mobile, Alabama

CASE PRESENTATION

History

A 58-year-old man presented with a 10-year history of slowly increasing muscle weakness. He had difficulty standing from a sitting or squatting position, climbing stairs, and getting in his truck as well as mild hand grip, arm, and neck weakness. He had no speech, swallowing, or breathing difficulty. He denied any muscle stiffness, spasm, or twitches, except for having infrequent episodes of deep ache pain in the thighs, and less frequently in the shoulders and upper arms. The patient was seen by a local neurologist 2 years earlier, and reportedly his needle electromyography (EMG) showed a mix of positive waves and myopathic motor unit action potentials (MUAPs). He had declined a muscle biopsy.

The patient’s past medical history included diabetes mellitus, treated with metformin, bilateral cataracts removed 5 years prior, and an appendectomy at age 34 years. He smokes half a pack of tobacco per day and drinks two to three beers daily. The family medical history is positive for diabetes and his father died at age 61 years secondary to heart disease. He also was reported to have had muscle weakness. The patient had a living paternal aunt di-agnosed with muscular dystrophy.

Examination

The patient’s general physical examination was unremarkable. The neurological examination revealed normal cognitive func-tioning. Cranial nerves II-XII were normal on examination, except for mild ptosis and subtle bilateral facial weakness. He had

no diplopia or delayed eye opening after forceful closure. He was noted to have mild atrophy of the glutei and sternocleidomastoid muscles bilaterally. He had difficulty rising from a sitting position without pressing on the chair arms. Muscle strength examination using the Medical Research Council rating scale revealed weak-ness of neck flexors 4/5, shoulder abductors 4+/5, biceps 4+/5, long finger flexors 4+/5, hip flexors 4/5, hip abductors 4+/5 bilat-erally, and ankle extensors 4+/5 and normal strength for all other remaining muscle groups. No muscle hypertrophy, fasciculations, stiffness, or hand grip myotonia were found; however, he was noted to have mild thenar muscle percussion myotonia. Tendon reflexes were symmetric rated 2+, and no pathological reflexes were elicited. Sensory testing, coordination, and gait examination were all normal.

Initial Differential Diagnosis

The patient presented with chronic, slowly progressive proxi-mal muscle weakness of the shoulder, hip girdle, and neck. The differential diagnosis in patients with proximal weakness should include muscular dystrophies; metabolic, congenital, or mitochondrial myopathies; and acquired myopathies, such as inflammatory, toxic, or endocrine. Other diagnoses, such as late-onset spinal muscular atrophy (SMA) and generalized my-asthenia gravis should also be considered. Motor neuron disease usually presents in a different anatomical distribution of muscle weakness, and it is not likely in this case given that the patient’s weakness remained mild after 10 years. An inherited autosomal dominant myopathy should be suspected, given that the patient’s father reportedly had muscle weakness and he has a living pater-nal aunt diagnosed with muscular dystrophy.

Page 22: NM Update Course II: Inflammatory Myopathies/Myotonic

22

MYOTONIC DYSTROPHIES

The predominantly proximal weakness suggests a diagnosis of muscular dystrophy (such as limb girdle muscular dystrophy), metabolic myopathy (such as adult onset acid maltase deficiency), myotonic dystrophy (or dystrophia myotonica) (DM), late onset SMA, or mitochondrial myopathy. Although adult onset acid maltase deficiency presents with a slowly progressive weakness over many years, it is usually associated with respiratory muscles weakness and with no clinical myotonia observed. Mitochondrial myopathy is a relatively static proximal weakness with poor endurance and can be associated with external ophthalmoplegia. It has a different inheritance pattern. Late onset SMA usually presents at an earlier age than this case, with insidious weakness and atrophy of the pelvic girdle and proximal leg muscles. It is followed by involvement of the shoulder girdle and upper arms muscles, associated with fasciculations and depressed tendon reflexes. Muscular dystrophy, especially DM, should be highly considered given that the patient has muscle weakness, diabetes, bilateral cataract, and percussion myotonia as well as a positive family history for weakness, diabetes, and heart disease. The classical myotonic muscular dystrophy type 1 (DM1) is unlikely with predominantly proximal weakness. However, the patient’s proximal weakness, percussion myotonia, associated systemic diseases, and family history are consistent with an autosomal dominant inheritance and are suggestive of myotonic muscular dystrophy type 2 (DM2), also called proximal myotonic myopa-thy (PROMM). Myasthenia gravis is not highly suspected given that the patient has no diplopia, ptosis bulbar weakness, increased fatigue, or weakness fluctuations.

Laboratory Tests and Electrodiagnostic Studies

Normal or negative laboratory studies included: blood count and differential, serum chemistry, thyroid studies, liver function tests, sedimentation rate, serum lactate, acetylcholine receptor antibod-ies, and hemoglobin A1C. The serum creatine kinase (CK) was 642 IU/L (normal < 210 IU/L).

Figure. Waxing and waning discharges in the form of positive waves (100 µV/20 ms)._________________________________________________________

Electrodiagnostic (EDX) studies were performed. A standard motor and sensory nerve conduction study and an F wave were all normal. Repetitive motor nerve stimulation of the ulnar and spinal accessory nerve at 3 Hz at rest and post-exhaustion exer-cise showed no neuromuscular junction abnormality. The needle EMG examination demonstrated runs of discharges with waxing and waning amplitude and frequency, in the form of positive waves more prominent in the distal muscles (Figure). In addition, the needle EMG demonstrated fibrillation potentials and frequent polyphasic MUAPs. Many of these were of short duration and small amplitude with early recruitment predominantly in the proximal limb muscles. Overall the EDX study was characteristic of myotonia with associated myopathic features predominantly in the proximal muscles, supportive of the clinical diagnosis of

DM2. The diagnosis of DM2 was confirmed by a positive genetic test for expanded tetranucleide repeats mutation in the zing finger protein 9 (ZNF9) on chromosome 3.

DISCUSSION

Myotonic Dystrophy

The myotonic disorders are a heterogeneous entity, generally divided into dystrophic and nondystrophic groups in which myo-tonia might be the single common clinical feature. The nondys-trophic myotonias can be further subdivided based on the type of muscle channel involved and the presenting features. Myotonic dystrophy is overwhelmingly the most common disease among the myotonic disorders and it is considered the most common form of muscular dystrophy in adults. The disease was first recognized by Steinert in 1909. It is inherited in an autosomal dominant manner of variable penetrance, anticipation, and a ma-ternal transmission in the congenital form.1,2 Although classified as a muscle disease with predominant myotonia and progressive weakness, DM is clearly a disease of multisystemic nature owing to variable ocular, cardiac, endocrine, reproductive, gastrointesti-nal, skin, and cognitive involvement. The disease is classified into DM1 and DM2. There may be more genetically distinct forms yet to be identified.

Myotonic Dystrophy Type 1

Myotonic dystrophy type 1 is the most common form of muscu-lar dystrophy, with an incidence of 13.5/100,000 based on Swiss data, and an estimated prevalence of around 5/100,000 for most European and North American populations.3 It is the most severe form among the myotonic disorders. Great variability of clini-cal expression exists, ranging from the severe congenital form to subclinical cases. Although DM1 can present at any age, in the majority of cases onset of symptoms and diagnosis occurs in the late teens or early adulthood. The most common present-ing complaint is usually muscle weakness. Myotonia is often a “minor nuisance” compared with other symptoms. Less com-monly, patients with DM1 may initially present with subnormal intelligence; premature cataracts; and pulmonary, endocrine, or gastrointestinal symptoms. Neck flexor and distal limb muscle weakness usually starts early in the disease course, especially in the forearm and finger flexors, and in the ankle dorsiflexors. It then slowly progresses to affect the proximal muscles, often asso-ciated with muscle atrophy.4 Facial features including mild ptosis, weakness of the eyes and mouth closure, frontal balding, a long face with hollowed cheeks, and temporalis and masseter wasting are other characteristics. Action myotonia, such as delayed re-laxation of the fingers following a forceful hand grip, is a minor complaint, aggravated by cold weather. It will typically diminish with repeated attempts (“warm-up” phenomenon). Often patients deny having myotonia or appear to be unaware of it and it has to be actively elicited by percussion of the thenar eminence, the tongue, or during a needle EMG.

The extramuscular manifestations in DM patients include a number of frequent multisystemic clinical features. Posterior subcapsular cataracts of variable severity and lenticular opaci-

Page 23: NM Update Course II: Inflammatory Myopathies/Myotonic

23

PRINCIPLES OF NCS AND NEEDLE EMG

ties are invariably present in patients over the age of 20 years. These are best identified by slit lamp examination.5 Cardiac con-duction defects are common and seen in up to 90% of patients. Cardiomyopathy is rare and the cardiac abnormalities do not cor-relate with the severity of muscle dysfunction.6 With prolonged disease duration, fatal arrhythmias and sudden death may occur in DM1 patients.7 Insulin insensitivity, hyperglycemia, and hyper-insulinemia following a glucose tolerance test are common. The incidence of overt diabetes mellitus is not increased.8 Testicular failure, low serum testosterone, and impotence in males are seen in both DM1 and DM2.9 Cognitive and neuropsychological testing in DM frequently reveals mild cognitive deficits, poor insight, and behavioral and personality disorders.10 Magnetic resonance imaging nonspecific abnormalities of cerebral white matter and abnormal positron emission tomography have been reported.11 Smooth muscle involvement of the gastrointestinal tract may cause dysphagia, decreased motility, constipation, and fecal incontinence.12 Diaphragmatic dysfunction may account for the alveolar hypoventilation, frequently seen in DM patients. It is more striking in infants with the congenital form. Sleep apnea, hypersomnia, and disrupted sleep are also common.13

Myotonic Dystrophy Type 2 (or Proximal Myotonic Myopathy)

Myotonic dystrophy type 2 was first identified in 1994 as an au-tosomal dominant disease, similar to DM1 in most of the clinical features, including muscle weakness, myotonia, and multisystem involvement. However, it has a characteristic pattern of muscle weakness and is genetically distinct from DM1.14,15 The incidence of DM2 is uncertain, but it is seemingly less common than DM1. The age of onset is usually between 20 and 60 years, although it may start at an earlier age. Anticipation is much milder than in DM1, and congenital forms have not been described.16 The disease may present with intermittent disabling pain and stiffness in the thighs, shoulders, or upper arms. The weakness is predomi-nantly proximal and progresses to involve the neck flexors, long finger flexors, elbow extensors, hip flexors, and knees extensors. The main complaint of most affected patients is hip girdle and proximal lower limbs weakness. The weakness may fluctuate and/or worsen during pregnancy. Clinical myotonia is variably present and may be missed. It is associated with “warm-up” phe-nomena. In a case report it was evident only during pregnancy. However, in a large series electrical myotonia was demonstrable in the majority of patients.17,18 Cognitive and systemic abnormali-ties are generally milder than in DM1, and fatal cardiac arrhyth-mias or sudden death rarely occur.

Congenital Myotonic Dystrophy

Congenital myotonic dystrophy was the first myotonic disorder to be recognized. It is a distinctive and potentially fatal form of DM, with the affected parent always being the mother. Hydramnios and reduced fetal movements in the later part of pregnancy are common. The principal clinical features of the disease are rec-ognized after birth. It is unusual for all features to be present in a single patient. Major clinical features include: bilateral facial weakness in over 85% of cases, accompanied by a “tented” upper lip, prominent jaw muscle weakness, hypotonia, and feeding and sucking difficulty of varying severity.19,20 The primary causes of

mortality in affected infants are likely bronchial aspiration and neonatal respiratory distress due to diaphragmatic weakness and hypoplasia and intercostal muscle weakness.21 Talipes may occur in about half the cases and generalized arthrogryposis is reported in a small portion of patients.22 Myotonia is absent in infancy and becomes evident later in childhood. Surviving infants show delayed motor and speech development, swallowing difficulty, and mild-to-moderate cognitive deficits. Once adolescence is attained, the disease follows the same course as the later onset disease. The characteristic facial features and eliciting myotonia in the mother allow instant recognition of the diagnosis in many cases. Although the prenatal diagnosis of DM is readily accom-plished by testing for cytosine-thymine-guanine (CTG) repeats in the amniotic fluid or chorionic villi, it is not possible to predict whether a fetus with an expanded mutation will have either con-genital or later onset DM.

Genetics and Pathogenesis

Myotonic dystrophy type 1 is caused by an expansion of CTG tri-nucleotide repeats in the myotonin protein kinase (DMPK) gene on chromosome 19q13.2.23 The number of the CTG repeats may expand with the next generation due to anticipation phenomena and correlates with the muscle dysfunction severity.24 Myotonic dystrophy type 2 is caused by an expansion of a CCTG tetranu-cleotide repeat within intron 1 of the ZNF9 gene on chromosome 3. This very large repeat accumulates as RNA foci in the nuclei of the affected tissue.25 Muscle weakness and the multisystemic clinical features common to both DM1 and DM2 are caused by the toxic gain effect on cells subsequent to these repeat expan-sions in RNA.26

Diagnosis

Myotonic dystrophy diagnosis can be achieved on clinical grounds and with needle EMG. Genetic testing confirms typical or atypical presentation and is commercially available for both DM1 and DM2. Minors should not be tested unless symptomatic or a diagnosis is necessary. Prenatal testing via amniocentesis or chorionic villous sampling assesses fetal risk in diagnosed parents. Serum CK may be normal or mildly elevated. Standard motor and sensory conduction studies are normal. Needle EMG demonstrates myotonic discharges often elicited by needle movements or voluntary contraction. Although at times several muscles may need to be tested before discharges can be found, they are less abundant in DM2. Diffuse fibrillations and positive waves, often masked by the myotonic discharges (as well as myopathic MUAPs and early recruitment), are detected in weak distal and/or proximal limb muscles. Electrical myotonia may not be elicitable in infants with the congenital form of DM, but it can be easily demonstrable in the mother’s hand muscles. Muscle biopsy, which is not required for diagnosis, shows characteristic type I fiber atrophy and occasional ring-band fibers in DM1. Type II fiber atrophy and pyknotic nuclei are common in DM2 with abundant central nuclei and intranuclear RNA inclusions. Necrotic muscle fibers are uncommon findings and increased connective tissue is usually mild. This is mostly seen in the late stages of the disease.27

Page 24: NM Update Course II: Inflammatory Myopathies/Myotonic

24

MYOTONIC DYSTROPHIES

Therapy and Prognosis

There is no known effective treatment for muscle weakness or curative therapy for the multisystemic manifestations in DMs. Small clinical trials of creatine monohydrate and recombinant human growth hormone and controlled trials of dehydroepi-androsterone sulfate in DM1 patients showed no efficacy.28 Therapies are primarily focused on drug treatment of myotonia and muscle pain, along with management of the various systemic manifestations. Patients with DM rarely complain of myotonia, and drug management should be limited to those with bother-some myotonia due to the potential cardiac conduction side effects. Quinine sulfate 0.5 g twice daily, procainamide 0.5 three times daily, or tocainide 400 mg three times daily are effective treatments for myotonia, but they potentiate cardiac arrhythmias. None of these drugs should be used in the presence of any cardiac disturbances. Mexiletine 200 mg three times daily, with electro-cardiogram (EKG) monitoring, or phenytoin 100 mg three times daily are helpful in relieving disabling myotonia.29 Muscle pain is usually secondary to weakness and deconditioning and can be treated with nonsteroidal anti-inflammatory drugs or other ap-propriate pain medications.

General management includes careful cardiac monitoring for conduction disturbances including screening with yearly EKGs. Cardiology referral, Holter monitoring, and echocardiograms are necessary if EKGs show conduction defect or arrhythmias. In patients with second or third degree heart block or in those for whom there is concern of unpredictable sudden death, pacemaker placement might be required. Pulmonary function tests and pre-cautions to avoid respiratory problems, including increased risk of aspiration and pneumonia, are recommended. Noninvasive ventilatory assistance (bilevel positive airway pressure or contin-uous positive airway pressure) might be required in patients with clinically-significant hypoventilation, diaphragmatic weakness, and sleep apnea. Ophthalmologic evaluation and cataract surgery might be needed. Orthopedic measures might be needed in chil-dren with talipes and other joint deformities. Orthotic devices for patients with foot drop help to facilitate their gait. Swallowing assessment in patients with dysphagia is important to avoid risk of aspiration, pneumonia, and poor nutrition. Speech therapy in patients with dysarthria is helpful. Endocrine status and meta-bolic function assessment in symptomatic patients or those sus-pected with dysfunction is necessary. Anesthetists and surgeons should be made aware of the hazards associated with anesthesia and surgery in patients with DMs. Nonstrenuous exercise is safe and may improve physical fitness and endurance. Genetic coun-seling is essential and of particular value for individuals with an affected parent. It is also essential for those affected individuals who wish to have children or to have a prenatal diagnosis.

Most DM patients remain mobile throughout their life and are usually free from the contractures seen in most disabling neuro-muscular disorders. It is important to recognize that life expec-tancy in DM1 patients is significantly reduced due to increased risk of fatal arrhythmias with sudden death, respiratory disease, neoplasia, and coronary artery disease. Longitudinal and cohort studies showed 7.3 times higher mortality rate than the age-matched control subjects with a mean age at death of 53 years.30,31

Myotonic dystrophy type 2 has rarely been associated with fatal arrhythmias or respiratory insufficiency, and the congenital form has not been identified in DM2 cases.

REFERENCES

1. Steinert H. Myopathologische beitrage 1, Uber das klinische und anatomische bild des muskelschwundes der myotoniker. Dtsch Z Nervenhlk 1909;37:58-104.

2. Magee AC, Hughes AE. Segregation distortion in myotonic dystrophy. J Med Genet 1998;35:1045-1046.

3. Emery AEH. Population frequencies of inherited neuromuscular dis-eases, a world survey. Neuromuscul Disord 1991;1:19-29.

4. Thornton C. the myotonic dystrophies. Semin Neurol 1999;20:25-33.5. Burian HM, Burns CA. Ocular changes in myotonic dystrophy. Am J

Ophthalmol 1967;63:22.6. Phillips MF, Harper PS. Cardiac disease in myotonic dystrophy.

Cardiovasc Res 1997;33:13-22.7. Sabovic M, Medica I, Logar N, et al. Relation of CTG expansion and

clinical variables to electrocardiogram conduction abnormalities and sudden death in patients with myotonic dystrophy. Neuromuscular Disord 2003;13(10):822-826.

8. Moxley RT, Griggs RC, Goldblatt D. decreased insulin sensitivity of forearm muscles in myotonic dystrophy. J Clin Invest 1978;62:857-867.

9. Day JW, Roelofs R, Leroy B, et al. Clinical and genetic characteristics of a five-generation family with a novel form of myotonic dystrophy; DM2. Neuromuscl Disord 1999;9:19-27.

10. Delaporate C. Personality patterns in patients with myotonic dystrophy. Arch Neurol 1998;55:635-640.

11. Hund E, Jansen O, Koch MC, et al. Proximal myotonic myopathy with MRI white matter abnormalities of the brain. Neurology 1997;48:33-37.

12. Lecointe-Besancon I, Leroy F, Devroed G, et al. A comparative study of esophageal and anorectal motility in myotonic dystrophy. Dig Dis 1999;44:1090-1099.

13. Coccagna G, Mantouant M, Parch C, et al. Alveolar hypoventila-tion and hypersomnia in myotonic dystrophy. J Neurol Neurosurg Psychiatry 1975;38:977.

14. Ricker K, Koch MC, Lehhmann-Horn F, et al. Proximal myotonic myopathy; a new dominant disorder with myotonia, muscle weakness, and cataracts. Neurology 1994;44:1448-1452.

15. Ricker K, Koch MC, Lehhmann-Horn F, et al. Proximal myotonic my-opathy, clinical features of a multisystemic disease similar to myotonic dystrophy. Arch Neurol 1995;52:25-31.

16. Schneider C, ZieglerA, Ricker K. Proximal myotonic myopathy. Evidence for anticipation in families with linkage to chromosome 3q. neurology 2000;55:383-388.

17. Newman B, Meola G, O’Donovan DG, et al. Proximal myotonic myopathy (PROMM) presenting as myotonia during pregnancy. Neuromuscul Disord 1999;9:144-149.

18. Day JW, Ricker K, Jacobsen JF, et al. Myotonic dystrophy type 2. Molecular, diagnostic, and clinical spectrum. Neurology 2003;60:657-664.

19. Hageman AT, Gabreels FJ, Liem KD, et al. Congenital myotonic dys-trophy; a report on thirteen cases and a review of the literature. J Neurol Sci 1993;115:95-101.

20. Harper PS. Congenital myotonic dystrophy in Britain, clinical aspects. Arch Dis Child 1975;50:505.

21. Bossen EH, Selburne JD, Verkauf BS. Respiratory muscle involvement in infantile myotonic dystrophy. Arch Pathol 1974:97:250.

22. Dyken PR, Harper PS. Congenital dystrophia myotonica. Neurology 1973;23:465.

23. Meola G. Genetic and clinical heterogeneity in myotonic dystrophies. Muscle Nerve 2000;13:1789-1799.

Page 25: NM Update Course II: Inflammatory Myopathies/Myotonic

25

PRINCIPLES OF NCS AND NEEDLE EMG

24. Harper PS, Harley HG, Reardon W, et al. Review article: anticipation in myotonic dystrophy; New light on an old problem. Am J Hum Genet 1992;51:10-16.

25. Liquori CL, Ricker K, Moseley ML, et al. Myotonic dystrophy type 2 caused by a CCTG expansion in intron 1 of ZNF9. Science 2001;293:864-867.

26. Day JW, Ranum LP. RNA pathogenesis of the myotonic dystrophies. Neuromuscl Disord 2005;15(1):5-16.

27. Vihola A, Bassez G, Meola G, et al. Histopathological differences of myotonic dystrophy type 1 (DM1) and PROMM/DM2. Neurology 2003;60(11):1854-1857.

28. Tarnopolsky M, Mahoney D, Thompson T, et al. Creatine monohydrate supplementation does not increase muscle strength, lean body mass, or muscle phosphocreatine in patients with myotonic dystrophy type 1. Muscle Nerve 2004;29(1):51-58.

29. Griggs RC, Davies RJ, Anderson DC, et al. Cardiac conduction in myotonic dystrophy. Am J Med 1975;59:37.

30. de Die-Smulders CE, Howeler CJ, Thijs C, et al. Age and causes of death in adult-onset myotonic dystrophy. Brain 1998;121:1557-1563.

31. Mathieu J, Allard P, Potvin L, et al. A 10-year study of mortality in a cohort of patients with myotonic dystrophy. Neurology 1999;52:1658-1662.

Page 26: NM Update Course II: Inflammatory Myopathies/Myotonic

2626

Page 27: NM Update Course II: Inflammatory Myopathies/Myotonic

27

NEUROMUSCULAR UPDATE II

27

CASE PRESENTATION

History

A 60-year-old woman presents for evaluation of muscle stiffness and pain since age 7. She recalls that during winter in elementary school she would not be able to unzip her coat without the teacher’s help. In addition to the cold weather, her stiffness is worse with any repeated activity or exercise. She reports difficulty shoveling snow from the driveway and pulling weeds in the garden. She states, “I learned to live with it all these years, but lately I feel worse.” She describes fatigue and aching muscle pain affecting her arms and thigh muscles. Her past medical history includes remote history of idiopathic thrombocytopenic purpura, hypertension, and Hashimoto’s thyroid disease; she had hysterectomy and right carpal tunnel surgery. Her medications include hydrochlorothiazide, cozaar, atenolol, levothyroxine, and trazodone. She is allergic to sulfa and amoxicillin. She retired as a flight attendant and moved to Ohio recently. She has no siblings. She reports no similar history in the family, including her son and her two grandchildren.

Examination

She is afebrile and blood pressure is normal. Mental status is normal and language is intact. Her cranial nerves examination is unremarkable except for eyelid myotonia that worsens with repeated testing. She has normal muscle strength, 2+ and symmetric muscle stretch reflexes, and down going plantar responses. She has normal pin prick and vibratory sensation. Furthermore, she has hand-grip myotonia that worsens with repeated trials. She has no frontal balding, temporalis or masseter

muscles atrophy, cataract, or thinning of the neck muscles.

Laboratory Studies

Serum creatine kinase (CK), complete blood count, electrolytes, liver function, and thyroid-stimulating hormone are all unremarkable. Electrodiagnostic (EDX) testing reveals normal sensory and motor nerve conduction studies. Needle electromyography (EMG) shows diffuse myotonic discharges.Differential DiagnosisThe differential diagnosis for a patient with muscle stiffness, pain, and cold-induced weakness includes:

• Nondystrophic myotonia• Myotonia congenita (MC)• Paramyotonia congenita (PC)• Sodium channel myotonias

• Myotonic dystrophyThe patient’s cardinal symptoms are muscle stiffness, pain, and cold-induced weakness since childhood. The presence of paradoxical myotonia (worsening with activity), eyelid myotonia, prominent upper limb and face involvement, extreme sensitivity to cold, and widespread myotonic discharges on needle EMG makes PC a likely diagnosis (see discussion). Specialized EDX testing (short exercise test) and genetic testing helped establish the diagnosis.

DISCUSSION

Myotonic disorders have in common a peculiar clinical phenomenon of delayed skeletal muscle relaxation following a trigger such as voluntary muscle contraction or percussion.1

Nondystrophic Myotonic Disorders

Bakri Elsheikh, MBBSAssociate Professor of Clinical Neurology

Director of the EMG LaboratoryWexner Medical Center at the Ohio State University

Columbus, Ohio

Page 28: NM Update Course II: Inflammatory Myopathies/Myotonic

2828

Clinical myotonia is associated with needle EMG recordings of runs of repetitive positive waves or fibrillation potentials representing repetitive muscle fiber action potentials. Typical myotonic discharges demonstrate waxing and waning of both amplitude and frequency, producing a peculiar sound that is compared to a dive bomber or a revving chain saw.2 Patients with myotonia usually report stiffness and difficulty relaxing muscles. Eyelid myotonia can lead to difficulty opening the eyes causing a squint that may last for minutes.1 Parents might report a child is having difficulty opening the eyes when crying. Lid lag phenomenon is noted with downward gaze after sustained upward gaze. The patient will appear as if staring or having a frightened look.1 Stiffness in the legs may cause tripping and falls.

On examination, action myotonia is elicited by having the patient forcefully close the eyes or make a tight fist while observing for difficulty relaxing the muscles. This is repeated successively three to four times. Usually repeated activity will improve the myotonia; a phenomenon referred to as “warm-up.” This is in contrast with paramyotonia (paradoxical myotonia) where repeated activity worsens the myotonia.

Percussion myotonia is evoked by tapping the thenar eminence or the posterior forearm (wrist extensor) muscles with a reflex hammer. Tongue myotonia is elicited by tapping the tongue lightly between two tongue blades while observing for a contraction resembling a ring.1

Myotonic disorders are divided into myotonic dystrophies and nondystrophic myotonias. Myotonic dystrophies were described earlier in this course. The term “nondystrophic myotonias” is used to refer to an inherited group of skeletal muscle channelopathies characterized by muscle stiffness and EDX evidence of myotonic discharges.2 In contrast to the myotonic dystrophies, this group of disorders lack progressive weakness and extramuscular manifestations (cardiac, ocular, respiratory, gastrointestinal, and endocrine). Occasionally, differentiating an early dystrophy, particularly myotonic dystrophy type 2, from nondystrophic myotonia can be challenging.3 Subgroups of nondystrophic myotonia are identified based on the clinical and EDX phenotype, channel involved, and genetic mutation (Table). The advances in the understanding of the molecular and genetic pathogenesis have helped to simplify the approach to patients with these disorders. The clinical phenotype and EDX pattern are useful in determining the channel involved and thus guide the genetic testing.

The nondystrophic myotonias are divided into two major groups based on the type of ion channel involved: chloride and sodium channelopathies. The chloride channelopathies include patients with autosomal recessive (AR) MC (Becker’s disease) and patients with autosomal dominant (AD) MC (Thomsen’s disease).The sodium channelopathies encompass two groups of disorders: PC and sodium channel myotonias (SCMs). In addition to the muscle stiffness, PC is characterized by concomitant episodes of extreme cold sensitivity and cold-induced weakness. This helps differentiate it from the sodium channel myotonias, a group that shares lack of cold sensitivity and potassium aggravation, hence the name “potassium aggravated myotonia.” Several subtypes (myotonia fluctuans, myotonia permanens, and acetazolamide responsive myotonia) were described based on the clinical

phenotype. Subsequently, sodium channel mutations were identified in patients with all the clinical phenotypes.

In addition to the primary myotonic disorders described above, other disorders with occasional myotonia were described. These are primarily myotonic discharges seen on needle EMG without clinical myotonia. Examples include acid maltase deficiency, hypothyroidism, and inflammatory myopathies as well as the effects of medications such as colchicine, statins, and clofibrate.2 However, it should be noted that in some of these cases, complex repetitive discharges might be mistaken for myotonic discharges.5

Disease Myotonia congenita

Myotonia congenita

Paramyotonia congenita

Sodium channel myotonia

Eponyms Becker’s disease

Thomsen’s disease

Eulenburg’s disease

Potassium aggrivated myotonias*

Inheritance AR AD AD AD

Gene CLCN1 CLCN1 SCN4A SCN4A

Age at onset Late childhood

Infancy and early childhood

First decade of life

Myotonia Legs > arms Arms > legs ± face

Arms and face > legs

Arms and face > legs

Repeated actvity Warm-up Warm-up Paradoxical

myotonia

Warm-up and paradoxical myotonia

Cold sensitivity

Absent or mild Absent or mild Extreme Absent to severe

Weakness

Transient weakness with initiation of movement

Rare

Cold- or exercise-induced weakness is common

Absent

Eyelid myotonia Rare Rare Common Common

Muscle pain Rare Rare Common Common

Muscle hypertrophy Common Common Less common Less common

Serum CK Normal-to-mild elevation

Normal-to-mild elevation

Mild-to-moderate elevation

Mild elevation

Needle EMG

Myotonic discharges profuse and widespread

Myotonic discharges less prominent

Myotonic discharges profuse and widespread

Myotonic discharges profuse and widespread

Chloride Channelopathies

Myotonia Congenita

Myotonia congenita is the most common skeletal muscle ion channelopathy. It is caused by mutation in the CLCN1 gene on chromosome 7q35, which encodes the skeletal muscle chloride channel 1.6 Chloride control resting membrane potential and impairment of chloride conductance in patients with MC shift

NONDYSTROPHIC MYOTONIC DISORDERS

Page 29: NM Update Course II: Inflammatory Myopathies/Myotonic

29

NEUROMUSCULAR UPDATE II

29

the membrane potential towards hyperexcitability. There are two major types of MC: an AD form (Thomsen’s disease) described in the 19th century (1876) by Julius Thomsen (who shared the disease with multiple members of his family)7 and an AR form (Becker’s disease) which was described decades later (1966) by P.E. Becker.8,9 Contrary to old data showing the AD type to be more common, recent studies suggests the reverse, with the AD type representing around 37% of the mutation-positive persons.4

Usually, patients will have myotonia and stiffness present from birth, however symptoms may not develop until childhood. Muscle stiffness is enhanced by cold and inactivity and relieved by exercise. Muscle strength is typically normal. Transient weakness is described and it usually last few seconds. Muscle hypertrophy, sometimes pronounced, giving a muscular physique and athletic appearance, is a common feature. Overall, muscle pain is underestimated in myotonic disorders; it is reported in 28% of MC patients.10 Myotonic symptoms tend to improve with age, but they do not disappear.

In general, the two types of MC (recessive and dominant) share many clinical features but some are different (Table). The AR type is usually more severe, primarily affecting the legs, with more frequent muscle hypertrophy and transient weakness with initiation of movement.3 The AD form has an earlier age of onset and affects mainly the arms.

Serum CK is normal or slightly elevated. Nerve conduction study usually shows normal sensory and motor conductions. Needle EMG shows profuse and widespread myotonic discharges. Muscle biopsy is not needed and if performed it is usually normal or shows mild nonspecific changes. The short exercise test, a measure of muscle membrane excitability, involves comparing compound muscle action potential (CMAP) response at rest with CMAP responses following a series of 10 s of exercise. A baseline CMAP is recorded at rest from the abductor digiti minimi (ADM) muscle after which the patient is asked to perform a maximal isometric contraction of the ADM muscle and then relax completely. Postexercise CMAP recordings are obtained in 10-s intervals for 1 min. The amplitudes of the postexercise responses are compared to the baseline response.11 A decrease by > 20% is considered abnormal. The test is repeated three times at normal temperature and after cooling to help identify various specific patterns.11,12

There are three patterns associated with nondystrophic myotonias, the most common or which in MC patients is pattern II. It shows a drop in CMAP amplitude with a rapid recovery in 1 min. There is less decrement with repeated trials, reflective of the clinical “warm-up” phenomenon. Furthermore, there is no additional effect to cooling. This pattern is more common in the AR MC patients but might be seen in other diseases with chloride channel involvement, including AD MC and in myotonic dystrophy patients.3 Some of the AD MC patients manifest pattern III, which reflects a normal response or lack of an abnormal pattern. A recent study found no additional benefit to performing 10 Hz repetitive nerve stimulation.13 Commercially-available genetic testing helps establish the diagnosis (www.genetests.org). There are more than 120 described chloride channel mutations, the majority of which are associated with AR MC. Furthermore, for reasons that are not clear, a specific mutation can result in AR MC in one family and AD MC in other families. Sequencing of the whole gene is needed, as one of the pitfalls for genetic testing is the presence

of many uncommon pathogenic mutations. Thus, one should be cautious interpreting commercial testing limited only to common mutations. Also, a recent trial showed 6% of a MC cohort with a single recessive mutation has an exon deletion or duplication.14

It is important to counsel patients on the effect of the activity and cold exposure. Genetic counseling should be offered as well. Many patients might not require drug treatment for the myotonia. However, those with disabling symptoms will benefit from therapy. A Cochrane Review concluded a lack of good-quality data to allow for specific recommendations.15 Many specialists consider mexiletine, an anti-arrhythmic drug, as the drug of choice for treatment of myotonia. It is proven safe and well tolerated in a recent study by the Consortium for Clinical Investigation of Neurologic Channelopathies. The data was presented at the 2012 American Academy of Neurology annual meeting showing that mexiletine at a dose of 200 mg three times daily is effective therapy for symptoms and signs of myotonia in patients with nondystrophic myotonias.16 Because of the drug’s potential to cause prolonged QT and cardiac arrhythmia, an initial electrocardiogram and close cardiac monitoring for prolonged QT are needed. Other side effects include gastrointestinal, dizziness, and tremors. Other medications used with variable results are flecainide, dilantin, and carbamezapine.

Sodium Channelopathies

Paramyotonia Congenita

Paramyotonia congenita is an AD disease caused by a mutation in the SCN4A gene on chromosome 17q23, which encodes the alpha subunit of the voltage-gated sodium channel.17 It is allelic with hyperkalemic periodic paralysis, hypokalemic periodic paralysis type 2, and sodium channel myotonias. Though myotonia is seen in 75% of hyperkalemic periodic paralysis patients, the most prominent clinical picture is weakness.18 Described by Eulenberg in the 19th century,19 PC usually manifests in the first decade of life. The myotonia is more prominent in the arms, face, and tongue. Muscle hypertrophy is less common compared to patients with MC. Some of the main features that help differentiate PC from MC are paradoxical myotonia (worsens with repeated activity), extreme cold sensitivity, and cold-induced weakness lasting up to few hours (Table). Some patients complain of muscle pain, though it is usually mild. In general, clinical features are helpful in differentiating the various types of nondystrophic myotonias but there is significant overlap and thus mutation analysis is required to reach a specific diagnosis.

Serum CK is usually normal or mildly elevated. Needle EMG shows myotonic discharges. A recent study described other discharges, including dolphin-sounding low amplitude (100-600 μV), high frequency musical 150-250 Hz discharges in large limb muscles.13 Also higher amplitude tornadeo-shaped, neuromyotonia-like discharges are noted in some patients.13 The short exercise test shows a gradual and relatively prolonged decrement after brief exercise, indicative of pattern I.11 The decrement increases with repeated trials and with cooling reflective of the clinical worsening noted with repeated activity and cold.12 Confirmation of the diagnosis is made by establishing a pathogenic mutation in the SCN4A gene (www.genetests.org). Patients should receive

Page 30: NM Update Course II: Inflammatory Myopathies/Myotonic

3030

counseling on the effect of exercise, repeated activity, and cold as well as genetic counseling. Mexiletine can be used for symptomatic treatment.

Sodium Channel Myotonia

Sodium channel myotonia is caused by a mutation in the SCN4A gene on chromosome 17q23, which encodes the alpha subunit of the voltage-gated sodium channel. The disorder is allelic with PC, hyperkalemic periodic paralysis, and hypokalemic periodic paralysis type 2. It encompasses a group of various disorders, including myotonia fluctuans, myotonia permanens, and acetazolamide-responsive myotonia.20-23 All share the tendency to worsen with potassium ingestion and lack of cold sensitivity. In addition, other characteristics that help differentiate the group from MC patients are a delayed onset myotonia after exercise (10-30 min) and a more frequent occurrence of eye closure myotonia and muscle pain.3,10 Furthermore, transient weakness at initiation of movement reported with MC and cold-induced weakness reported with PC are not seen.3 Similar to other nondystrophic myotonia types, the clinical features are helpful in differentiating the subtypes; however, there is overlap, making genetic confirmation necessary.

Serum CK is normal to mildly elevated. Needle EMG showed diffuse myotonic discharges. The short exercise test shows a normal response, indicative of pattern III. Patients should receive genetic counseling and advice to avoid food rich in potassium. Mexiletine can be used for symptomatic treatment. Acetazolamide is helpful in some patients.

CASE PRESENTATION FOLLOWUP

The patient short exercise test showed a progressive CMAP decline with repeated trials. Genetic testing confirmed a sodium gene mutation SCN4 (theronine > methionine; codon 1313, nucleotide 3938) establishing the diagnosis of PC in the patient. The patient received mexiletine; however, she had difficulty tolerating the medicine because of dyspepsia and dizziness.In retrospect, the patient’s old records indicate a diagnosis of possible neuromyotonia based on needle EMG findings. This is likely explained by the recently described, distinctive neuromyotonia-like discharges noted in some of the patients with PC.

REFERENCES

1. Griggs RC, Mednell JR, Miller RG. Periodic paralysis and myotonia. Evaluation and treatment of myopathies. Philadelphia: F.A. Davis;1995, pp 26-29, 318-354.

2. Amato A, Russel J. Nondystrophic myotonias and periodic paralysis. Neuromuscular Disorders, 1st ed. New York: McGraw-Hill; 2008, pp 655-680.

3. Matthews E, Fialho D, Tan SV, Venance SL, Cannon SC, Sternberg D, Fontaine B, Amato AA, Barohn RJ, Griggs RC, Hanna MG; CINCH Investigators. The non-dystrophic myotonias: molecular pathogenesis, diagnosis and treatment. Brain 2010;133:9-22.

4. Fialho D, Schorge S, Pucovska U, Davies NP, Labrum R, Haworth A, Stanley E,Sud R, Wakeling W, Davis MB, Kullmann DM, Hanna MG. Chloride channel myotonia:exon 8 hot-spot for dominant-negative interactions. Brain 2007;130:3265-3274.

5. Miller TM. Differential diagnosis of myotonic disorders. AANEM monograph #27. Muscle Nerve 2008;37:293-299.

6. Koch MC, Steinmeyer K, Lorenz C, Ricker K, Wolf F, Otto M, Zoll B, Lehmann-Horn F, Grzeschik K-H, Jentsch TJ. The skeletal muscle chloride channel in dominant and recessive human myotonia. Science 1992; 257:797-800.

7. Thomsen, J. Tonische Kraempfe in willkuerlich beweglichen Muskeln in Folge von ererbter psychischer Disposition: Ataxia muscularis? Arch Psychiat Nervenkr 1876;6:702-718.

8. Becker PE. Zur Genetik der Myotonien. In: Kuhn E. Progressive muskeldystrophie, myotonie, myasthenie. Berlin: Springer-Verlag; 1966. pp. 247-255

9. Becker PE. Myotonia congenita and syndromes associated with myotonia. Vol. III. Topics in human genetics. Stuttgart: Georg Thieme; 1977.

10. Trip J, Drost G, Ginjaar HB, Nieman FH, van der Kooi AJ, de Visser M, van Engelen BG, Faber CG. Redefining the clinical phenotypes of non-dystrophic myotonic syndromes. J Neurol Neurosurg Psychiatry 2009;80(6):647-652.

11. Fournier E, Arzel M, Sternberg D, Vicart S, Laforet P, Eymard B, Willer JC,Tabti N, Fontaine B. Electromyography guides toward subgroups of mutations in muscle channelopathies. Ann Neurol. 2004;56(5):650-661.

12. Fournier E, Viala K, Gervais H, Sternberg D, Arzel-Hézode M, Laforêt P, Eymard B, Tabti N, Willer JC, Vial C, Fontaine B. Cold extends electromyography distinction between ion channel mutations causing myotonia. Ann Neurol 2006;60(3):356-365.

13. Tan SV, Matthews E, Barber M, Burge JA, Rajakulendran S, Fialho D, Sud R,Haworth A, Koltzenburg M, Hanna MG. Refined exercise testing can aid DNA-based diagnosis in muscle channelopathies. Ann Neurol 2011;69(2):328-340.

14. Raja Rayan DL, Haworth A, Sud R, Matthews E, Fialho D, Burge J, Portaro S, Schorge S, Tuin K, Lunt P, McEntagart M, Toscano A, Davis MB, Hanna MG. A new explanation for recessive myotonia congenita: Exon deletions and duplications in CLCN1. Neurology. 2012;78(24):1953-1958.

15. Trip J, Drost G, van Engelen BG, Faber CG. Drug treatment for myotonia. Cochrane Database Syst Rev. 2006.

16. Statland J, Salajegheh M, Bundy B, Wang Y, et al. Phase II Therapeutic trial of mexiletine in non-dystrophic myotonia: secondary outcomes show improvement in symptoms and signs of myotonia. Platform presentation (S55. 005). 65th Annual Meeting of the American Academy of Neurology. New Orleans, Louisiana. April 2012.

17. Ptacek LJ, Trimmer JS, Agnew WS, Roberts JW, Petajan JH, Leppert M. Paramyotonia congenita and hyperkalemic periodic paralysis map to the same sodium-channel gene locus. Am J Hum Genet 1991;49: 851-854.

18. Miller TM, Dias da Silva MR, Miller HA, Kwiecinski H, Mendell JR, Tawil R,McManis P, Griggs RC, Angelini C, Servidei S, Petajan J, Dalakas MC, Ranum LP, Fu YH, Ptácek LJ. Correlating phenotype and genotype in the periodic paralyses. Neurology 2004;63(9):1647-1655.

19. Eulenburg, A. Ueber eine familiare, durch 6 generationen verfolgbare Form congenitaler Paramyotonie. Neurologisches Centralblatt 1886;12:265-272.

20. Trudell RG, Kaiser KK, Griggs RC. Acetazolamide responsive myotonia congenita. Neurology 1987;37:488-491.

NONDYSTROPHIC MYOTONIC DISORDERS

Page 31: NM Update Course II: Inflammatory Myopathies/Myotonic

31

NEUROMUSCULAR UPDATE II

31

21. Ricker K, Moxley RT III, Heine R, Lehmann-Horn F. Myotonia fluctuans: a third type of muscle sodium channel disease. Arch. Neurol. 1994; 51:1095-1102.

22. Lerche H, Heine R, Pika U, George AL, Jr, Mitrovic N, Browatzki M, Weiss T, Rivet-Bastide M, Franke C, Lomonaco M, Ricker K, Lehmann-Horn F. Human sodium channel myotonia: slowed channel inactivation due to substitutions for a glycine within the III-IV linker. J. Physiol 1993;470:13-22.

23. Colding-Jorgensen E, Duno M, Vissing J. Autosomal dominant monosymptomatic myotonia permanens. Neurology 2006;67:153-155.

Page 32: NM Update Course II: Inflammatory Myopathies/Myotonic

32

Page 33: NM Update Course II: Inflammatory Myopathies/Myotonic

33

VIGNETTE ONE

A 34-year-old man was evaluated for pain in feet. He described the sensation as if a hot towel were wrapped around his feet. There was no weakness. Review of systems revealed dry eyes, blurred vision, and lightheadedness. He was evaluated for intermittent abdominal pain and vomiting without nausea. He has also had episodes of erectile dysfunction.

His cranial nerve examination was normal. He has normal tone, bulk, and strength. Sensory examination revealed normal light touch, pinprick, vibration, and joint position sense. Reflexes were absent in ankles only. Cerebellar examination was normal.

Questions

1A. What is the clinical pattern of his symptoms and signs?

A. Distal sensory loss.

B. Sensory with upper motor neuron signs.

C. Severe propriceptive deficit.

D. Sensory and autonomic.

1B. What other information would be helpful?

A. History of diabetes.

B. Ethnic background.

C. Family history.

D. All of the above.

1C. What other studies would be MOST useful?

A. Genetic testing.

B. Cardiac echocardiogram.

C. Autonomic testing.

D. Serum immunofixation.

VIGNETTE TWO

A 43-year-old man, with a history of papillary thyroid cancer, devel-oped right arm abduction weakness after surgical resection. He was referred to the EMG Laboratory for evaluation. He has weakness in right abduction only. Sensory examination and reflexes were normal.

Neuromuscular Vignettes

Annabel K. Wang, MDAssociate Professor of Neurology

University of California, IrvineOrange, California

Gregory T Carter, MD, MSMedical Director,

Muscular Dystrophy AssociationRegional Neuromuscular Center,

Olympia, Washington

Andrew W. Tarulli, MDAssistant Professor, Neurology

Harvard Medical SchoolBoston, Massachusetts

Page 34: NM Update Course II: Inflammatory Myopathies/Myotonic

34

NEUROMUSCULAR VIGNETTES

Questions

2A. What is the cause of his weakness?

A. Spinal accessory neuropathy.

B. Axillary neuropathy.

C. Brachial plexopathy.

D. Cervical radiculopathy.

Routine nerve conduction studies (NCSs) and needle electromyography (EMG) of the right arm is normal.

2B. What other studies should be performed?

A. Spinal accessory nerve motor study.

B. Needle examination of the trapezius.

C. Needle examination of the sternocleidomastoid.

D. All of the above.

Lat Amp Dur Area Temp Lat diff

(ms) (mV) (ms) (mVms) (°C) (ms)

Wrist 3.5 12.7 6.2 48.3 33.5 70

Elbow 8.2 12.3 6.3 46.6 33.3 240

Wrist 3 8.4 7.8 27.1 33.3 70

Below elbow 7 7.6 7 23.6 33.7 235

Above elbow 9.5 7.2 7.1 23.6 33.8 160

Neck 2.5 3.1 14.8 26 33.2

Neck 2.2 7.3 15.2 72.2 33.1

Accessory (spinal) R to trapezius R

2.5

Accessory (spinal) L to trapezius L

2.2

4.7

Ulnar R to abductor digiti minimi (manus) R

3

4

2.5

Nerve and site

Dist

(mm)

Median R to abductor pollicis brevis R

3.5

CV

(m/s)

51

59

64

F-Wave studies

M-Lat F-Lat

(ms) (ms)

Median R 3.7 29.2

Ulnar R 3.1 30.6

Nerve

2C. What are the consequences of injury to the spinal accessory nerve?

A. Lateral and upward displacement of the scapula.

B. Drooping of the shoulder.

C. Asymmetric shoulder shrug.

D. All of the above.

Below are the results of the patient’s electrodiagnostic (EDX) studies.

Motor NCSs

Page 35: NM Update Course II: Inflammatory Myopathies/Myotonic

35

PRINCIPLES OF NCS AND NEEDLE EMG

Needle EMG Examination

Sensory NCSs

Onset lat Peak lat Amp Temp Dist

(ms) (ms) (μV) (°C) (mm)

Wrist 2.3 3.1 33 33.9

Wrist 1.9 2.7 23 34

Forearm 1.8 2.3 25 33.9

Elbow 2.2 2.8 18 33.8

Elbow 2.3 2.8 10 33.8

Elbow 2.3 2.8 15 33.3

Elbow 2.5 2.9 9 33.3

Medial antebrachial cutaneous.L to Forearm.L

2.5 130

Lateral antebrachial cutaneous.L to Forearm.L

2.3 130

Medial antebrachial cutaneous R to forearm R

2.3 130

Lateral antebrachial cutaneous R to forearm R

2.2 130

Radial R to anatomical snuff box R

1.8 100

Ulnar R to digit V (little finger) R

1.9 110

Median R to digit II (index finger) R

2.3 130

Nerve and site

Lat diff

(ms)

52

57

57

59

56

59

57

CV

(m/s)

Muscle Insertion

activity Fibs/PSW Fasc Other Amp Dur Poly Recruit

Deltoid R Normal None None Normal Normal None Normal

Biceps brachii R Normal None None Normal Normal None Normal

Triceps brachii R Normal None None Normal Normal None Normal

Pronator teres R Normal None None Normal Normal None Normal

First dorsal interosseous R Normal None None Normal Normal None Normal

Abductor pollicis brevis R

Normal None None Normal Normal None Normal

Trapezius R Normal 2+ None Normal 2+ Many Mark Decr

Sternocleidomastoid R Normal 2+ None Normal 2+ Many Mod Decr

Infraspinatus R Normal None None Normal Normal None Normal

C4 paraspinal R Normal None None

Spontaneous activity Volitional MUAPs

Rate Comment

Normal

Normal

Normal

Normal

Normal

Normal

Normal Sat Portentials

Normal Sat Portentials

Normal

Page 36: NM Update Course II: Inflammatory Myopathies/Myotonic

36

VIGNETTE THREE

A 66-year-old man was evaluated for progressive walking difficulty over several months. He noticed the onset of tingling in his toes. He felt like he was walking as if he was drunk. He had intermittent falls.

His examination revealed normal tone, bulk, and strength. Vibration was absent in toes and joint position sense was mildly reduced in toes. His reflexes were 3+ in the upper extremities and 2+ in the lower extremities. His toes were upgoing.

Questions

3A. What is the clinical pattern of his symptoms and signs?

A. Distal sensory loss.

B. Sensory with upper motor neuron signs.

C. Severe proprioceptive deficit.

D. Sensory and autonomic.

3B. Which conditions lead to subacute combined degeneration?

A. Nitrous oxide abuse.

B. Vitamin B12 deficiency.

C. Copper deficiency.

D. All of the above.

Laboratory and examination results showed the following:• EDX studies: Absent bilateral sural responses only.• C-spine magnetic resonance imaging: Mild degenerative changes.• Vitamin B12, homocysteine, and methylmalonic acid levels:

Normal.• Copper: 62 (normal 85-155) mcg/dL.• Ceruloplasmin: 30 (normal 22-58) mg/dL.• Zinc: 120 (normal 50-155) mcg/dL.3C. What other history would be helpful?

A. Gastric surgery.

B. Excessive zinc ingestion.

C. Malabsorptive enteropathies.

D. All of the above.

VIGNETTE FOUR

A 53-year-old man had insidious onset of difficulty speaking and swallowing. He complains of a hoarse voice, along with loss of muscle mass in all his limbs. He was originally told by another neurologist (who was not a member of the American Association of Neuromuscular and Electrodiagnostic Medicine) that he had amyotrophic lateral sclerosis (ALS) and had “only a few years to live.” He is referred to you for a second opinion and some management issues.

Physical examination showed diffuse muscle wasting and multiple fasciculations. You also note hyporeflexia and what appears to be an intention tremor. There was also diminished fine motor movement, and a “floppy head.”

Questions

3A. Which additional finding is he also MOST LIKELY to have?

A. Ashworth grade II spastic-ity.

B. Cogwheel rigidity in his extremities.

C. Gynecomastia.

D. Oculomasticatory myor-hythmia.

Hyperhydrosis and allodynia

The patient is not able to elevate the soft palate and has markedly atrophy and decreased tongue movement. Having recently passed the American Board of Electrodiagnostic Medicine ex-amination, you are confident in your needle EMG skills and decide to restudy this man. Needle EMG showed acute and chronic denervation with fas-ciculations; peripheral NCSs showed decreased amplitude of compound motor action poten-tials but no conduction slowing

NEUROMUSCULAR VIGNETTES

CERVICAL COLLARS IN NEUROMUSCULAR DISEASE

Individuals with neck extensor muscle weakness may complain of neck stiffness, heaviness, and fatigue in holding the head up, and they may also notice difficulties in keeping the head upright with unexpected movements. In the later stages, and in cases of severe muscle weakness, the head drops forward, the cervical spine is completely flexed, and the patient experiences severe neck pain and anterior muscle tightness.

Several different types of collars that can support the head, protect the weakened muscles, and prevent further deformity are available. For mild-to-moderate weakness a soft foam collar is usually recommended. Soft collars are usually comfortable and well-tolerated, and neck movements are limited to a certain degree.

For moderate-to-severe weakness, a semi-rigid collar, such as the traditional Philadelphia™ collar, is required. Patients find these collars very warm, may experience discomfort at points of contact and pressure over the trachea, or may feel confined. If the individual has a tracheostomy, a Miami-J®, Aspen, or Malibu collar that allows for anterior neck access are prescribed. Warmth, discomfort, and a sense of confinement are also common with these collars. The Headmaster, Executive, and Canadian collars have an open air design, but do not control as well for lateral instability as the Miami-J®, Aspen, or Malibu collars. In practice, the Headmaster collar is well accepted by many patients and is a compromise

Page 37: NM Update Course II: Inflammatory Myopathies/Myotonic

37

NEUROMUSCULAR UPDATE II

or block. However, you had great difficulty obtaining any sensory nerve action potentials.

4B. What is his LIKELY diagnosis?

A. Amyotorphic lateral sclerosis.

B. Multi-focal motor neuropathy.

C. Spinal muscular atrophy III (Kugelberg-Welander syndrome).

D. Spinobulbar muscular atrophy (Kennedy disease).

E. Guillain-Barré syndrome

4C. The patient has tremendous difficulty holding his head up and really wants help with this. What type of collar might work BEST in this situation?

A. Freeman or Headmaster type cervical orthosis.

B. Minerva brace.

C. Halo type orthosis.

D. The “Iron Maiden” neck stabilizer.

VIGNETTE FIVE

A 9-year-old boy presents to the neuromuscular clinic for evalu-ation and management. The family is from Mexico and the parents do not speak English. The boy speaks limited English. You are able to ascertain through an interpreter that he had an older brother who died of pneumonia when he was 18 years old. He also had a maternal uncle who died “very young” from some type of heart problem. The boy is now having progressive difficulties walking and is falling all the time at school now. On examination you note reflexes are 1+ in the arms and absent in the legs, with pinprick and vibratory sensation intact. The boy has a distinct waddle when he walks and is up on his toes.

Questions

5A. Based on the photograph above, what additional findings does he also have?

A. Hyperlorodotic positioning of the spine with heel cord tightness.

B. Severe pectus excavatum.

C. Kyphosis with a secondary curve less than 20 degrees.

D. Scoliosis with a curve that likely exceeds 50 degrees and forced vital capacity (FVC) is < 25% predicted value, which is not corrected with spinal instrumentation.

5B. Knowing that he has a serum creatine kinase level of 20,000 U/L at rest, what is the BEST next test to perform in order to arrive at a definitive diagnosis?

A. EDX evaluation including single fiber EMG.

B. Forearm ischemic exercise testing.

C. DNA evaluation.

D. Muscle biopsy through needle aspiration.

5C. One year later spinal radiographs reveal scoliosis with a primary curve exceeding 25 degrees and FVC is < 50% predicted value. What is the next management course?

A. Aggressive spinal manipulation to reduce the curvature.

B. Referral to a pediatric spine surgeon for spinal fusion and instrumentation.

C. Fitting with a custom molded spinal orthosis.

D. Proprioceptive neuromuscular stretching through a neurode-velopmental therapist.

5D. When is spinal instrumentation in boys like this indicated?

A. Any time before the age of skeletal maturity.

B. When the primary curve exceeds 25 degrees and FVC is < 50% predicted value.

C. When the primary curve is less than 25 degrees.

D. Never because scoliosis is not corrected with spinal instru-mentation.

E. Never because scoliosis is best corrected with bracing.

VIGNETTE SIX

A 28-year-old man presents with a 5-7 year history of slowly progressing weakness beginning in the distal limb muscles but now affecting his hands. He is starting to notice muscle wasting. He also remembers that as a child he was always clumsy or not very athletic.

He now notes that he is having great difficulty walking and he frequently trips due to catching his feet. Frequent ankle sprains and falls are also a problem for him.

On examination, his reflexes are absent throughout. He demon-strates 4/5 strength in most major muscle groups. After walking for several minutes he does start to drag his feet.

between providing maximal stability and comfort.

Patients with combined cervical and upper thoracic weakness may benefit from a cervical-thoracic orthosis, or a sterno-occipital mandibular immobilizer (SOMI). These devices provide maximum support, but they are more expensive, heavy, and may be difficult to don and doff. For severe or intractable cervical or trunk weakness, referral to an orthotist or biomedical engineer for a custom-made device may be necessary.

Page 38: NM Update Course II: Inflammatory Myopathies/Myotonic

38

Questions

6A. Which type of bracing modality is MOST LIKELY to improve his gait?

A. Custom fit shoe orthotics with metatarsal pads.

B. Bilateral double metal upright ankle-foot orthotics (AFOs).

C. Lightweight, low profile AFOs with dorsiflexion assist.

D. Bilateral long leg, double metal upright knee ankle-foot orthotics (KAFOs).

E. Bilateral universal foot orthotics (UFOs).

6B. Which factor is MOST LIKELY to contribute to loss of ambula-tion in a patient with peripheral neuropathy?

A. Progressive dyspnea.

B. Weakness in the quadriceps muscles (< 3/5 on manual muscle testing).

C. Severe atrophy in the cerebellum.

D. Loss of pain sensation.

E. Weakness (< 4/5) in tibialis anterior muscles.

6C. Which gait description BEST characterizes his walking?

A. Toe walking.

B. A steppage gait, where the patient lifts their legs in an ex-aggerated fashion to clear the feet off the ground.

C. A reverse Trendelenberg gait caused by gluteus medius weakness.

D. A shuffling gait due to impaired proprioception.

E. A VIP gait often noted in executives and politicians.

6D. Regarding orthotics in peripheral neuropathy, which statement is the MOST appropriate and correct?

A. Custom fit shoe orthotics with metatarsal pads are most helpful in terms of improving balance.

B. Bilateral KAFOs are generally too heavy and cumbersome to be of much use.

C. UFOs cannot be used outside of Area 51.

D. Bilateral double metal upright AFOs are often preferred by patients as they provide excellent medial-lateral ankle support and stability.

E. Upper extremity orthotics are easy to use and very helpful for patients with peripheral neuropathy.

VIGNETTE SEVEN

A 57-year-old woman is referred to your office with slowly pro-gressive weakness in both proximal and distal muscles. She is much weaker on the left side of her body with striking asymme-try. She has remarkable weakness of the wrist and finger flexors, disproportionate to that of her extensors (see image). Hence, she has loss of finger dexterity and grip strength.

Questions

7A. All of the following are likely additional symptoms, EXCEPT:

A. Dysphagia.

B. “Floppy neck” syndrome.

C. Myalgias and cramping.

D. Sensory and autonomic dysfunction.

NEUROMUSCULAR VIGNETTES

DEVICES TO ASSIST WITH WALKING: MOBILITY AIDS

Muscle weakness and resultant balance problems will eventually necessitate that a patient use an aid to enhance mobility and safety. The type of assistive device recommended is determined by the degree of lower extremity (LE) and trunk weakness and spasticity, range of motion and strength of the upper extremities (UEs), extent and rate of progression of the disease, acceptance of the aid by the individual, and economic constraints. In addition to taking into account which device will ensure optimal function and safety, the weight of the mobility aid is an important factor to consider in decisionmaking, as individuals with amyotrophic lateral sclerosis (ALS) experience muscle fatigue.

Canes

Canes are usually recommended in the early stages of ALS for mild LE weakness or balance problems, as they provide the least amount of support. A cane is carried in the hand opposite to the most affected leg, requires good UE strength, and can be used on stairs. The standard wooden cane is the least expensive, has a curved handle, and can be made shorter but not longer. An adjustable aluminum cane easily adapts for various heights and is lightweight. An aluminum cane may have a curved handle or an offset handle. An offset handle allows the weight to be directed over the cane tip when in contact with the floor, rather than anteriorly, as is the case with a curved handle. Quad canes are aluminum canes with four points of floor contact. Quad canes provide greater stability than straight canes, but all tips must be in contact with the ground for stability. The size of base can vary and these canes are heavier to lift. Aluminum and quad

Page 39: NM Update Course II: Inflammatory Myopathies/Myotonic

39

NEUROMUSCULAR UPDATE II

7B. She continues to have slow but relentless progression of muscle weakness and is now having diffi-culty with ambulation and frequent falls. What one examionation/test should you be sure to perform?

A. Bone density scan to assess for osteoporosis.

B. Hemoglobin A1c blood test to rule out diabetes.

C. Opthamology examination to assess for cataracts.

D. Hepatobiliary iminodiace-tic acid scan to assess for gall bladder sludging.

7C. What is the MOST LIKELY cause of her difficulty in walking?

A. Weakness in her spinal ex-tensor musculature.

B. Loss of proprioception.

C. Weakness in her quadriceps muscles.

D. Loss of full field of vision.

7D. You want to prescribe some-thing to assist her in walking. What would be BEST to reduce her risk of falling?

A. Single point cane.

B. Front wheeled walker.

C. Quad (four-point) cane.

D. A hemi walker.

VIGNETTE EIGHT

A 40-year-old woman presents with 2 weeks of burning and

tingling pain in her fingers and toes followed by progressive leg weakness. She also has a 2-year history of repetitive vomiting and diffuse abdominal pain. On examination, she has severe distal lower extremity weakness with bilateral foot drop, distal sensory loss in a stocking-glove pattern, and absent deep tendon reflexes. Her skin is diffusely hyperpigmented. Laboratory studies are sig-nificant for a hematocrit of 28% with a mean corpuscular volume of 85 fL. Basophilic stippling is present. Chest X-ray shows an

ill-defined hyperdensity overlying the gastric bubble. Nerve con-duction studies show symmetrically reduced peroneal and tibial CMAP amplitudes with normal conduction velocities, reduced sural SNAP amplitudes with normal conduction velocities, and normal NCSs in the arms. Needle EMG shows fibrillation poten-tials and reduced recruitment in tibialis anterior, extensor hallucis longus, and gastrocnemius, but is otherwise normal.

Questions

8A. Which of the following is the MOST LIKELY diagnosis?

A. Acute intermittent porphyria.

B. Arsenic intoxication.

C. Guillain-Barré syndrome.

D. Fabry’s disease.

E. Polyarteritis nodosa

8B. Which of the following statements concerning this patient is TRUE?

A. She is likely to have diffuse hair loss.

B. Her transaminases should be normal.

C. Mees’ lines (leukonychia) are a frequent finding.

D. Motor symptoms are more likely to precede sensory symptoms.

E. Recent fish consumption may result in an increase in the measured arsenic level.

8C. Which of the following is the MOST appropriate for this patient?

A. Chelation with dimercaprol.

B. Gastrointestinal lavage.

C. Hematin and glucose.

D. Intravenous immunoglobulin.

E. Prednisone.

VIGNETTE NINE

An 18-month-old girl (see photo) presents for evaluation of weak-ness. She has a previous history of neonatal hypotonia, with re-spiratory failure requiring intubation and developmental delay. On physical examination, she has a long face, bilateral facial weakness, open jaw, bilateral ptosis, and moderate distal-greater-than-proximal muscle weakness. Hand-grip and percussion myotonia are present.

canes come in a variety of styles and sizes of handgrips. Patients with hand muscle weakness may be better able to grip an enlarged or molded handle.

Crutches

In general, crutches are rarely recommended for individuals with ALS, as the patient must have very good UE and trunk strength as well as adequate balance to use them. If crutches are recommended, Loftstrand or Canadian crutches are preferred. These crutches consist of a single upright, a forearm cuff, and a handgrip; the hands can be freed for standing tasks without having to release the crutch.

Walkers

Walkers provide greater support than canes and crutches, but are more bulky and may be cumbersome in confined spaces. Various types of walkers are available (e.g., folding, with or without wheels, with brakes, with seating surfaces, etc.) and some walkers can be modified to suit the individual’s walking environment and needs; for example, they may be fitted with a basket, food tray, or forearm trough. Standard aluminum walkers are the least expensive, very stable, and can be adjusted for various heights. However, they must be picked up and lowered during ambulation, and patients may find them heavy to lift. Walkers with wheels do not need to be lifted and they roll forward easily, thus they are usually recommended for patients with ALS; however, the stability of wheeled walkers is decreased and they may move forward too quickly. Some walkers come equipped with brakes. Push down brakes secure a walker when the patient loads his or her weight on the walker and are preferred over squeeze type brakes for patients with hand weakness. Specialized wheeled walkers are the most expensive. They have large wheels that can move over a variety of terrains.

Page 40: NM Update Course II: Inflammatory Myopathies/Myotonic

40

Questions

9A. What is the MOST LIKELY diagnosis?

A. Facioscapulohumeral dystrophy.

B. Myotonic dystrophy type 1.

C. Myotonic dystrophy type 2.

D. Myotonia congenita.

E. Slow-channel syndrome.

9B. How did this patient MOST LIKELY inherit this disorder?

A. Autosomal dominant, from her father.

B. Autosomal dominant, from her mother.

C. Autosomal recessive, from her father.

D. Autosomal recessive, from her mother.

E. It is X-linked recessive.

9C. What is the LIKELY number of CTG repeats in the DMPK gene in this patient?

A. < 30.

B. 50.

C. 100.

D. 250.

E. 1,000.

VIGNETTE TEN

A 63-year-old man presents with left shoulder pain and atrophy. Two years ago, while lifting weights, he noticed the sudden onset of left shoulder pain without radiation. The pain did not improve on its own, so he began phys-ical therapy. After 20 sessions, his pain had improved, but he also noticed that his left deltoid was becoming atro-phic. Directed neurological examina-tion shows atrophy of the left deltoid. Strength is normal with the exception of left shoulder abduction which is 4/5

and left external shoulder rotation which is 5−/5. There were no reflex asymmetries. Sensation was minimally decreased over the lateral left shoulder. Needle EMG and NCSs show the following:

Sensory NCSs

NEUROMUSCULAR VIGNETTES

NP amp = negative peak amplitude, Dist = distance, Temp = temperature, Vel = velocity

Motor NCSs

Onset Peak NP amp Temp Dist Vel

(ms) (ms) (μV) (°C) (cm) (m/s)

1. Wrist Digit II 2.7 3.5 20.1 14 51.9

1. Wrist Digit V 2.35 3.65 14.3 32.4 12.4 52.8

Nerve/sites Recording site

Left median

Left ulnar

Lat Amp Area Temp Dist Vel

(ms) (mV) (mVms) (°C) (cm) (m/s)

1. Wrist APB 3.65 11.6 43.4 32 7

2. Elbow APB 7.95 11.6 41 22.4 52.1

1. Wrist APB 2.7 14.9 45.9 72. Below elbow APB 6.7 13.9 45.5 22.9 57.2

3. Above elbow APB 7.85 12.4 41.9 7.2 62.6

Nerve/sites Recording Site

Left median, APB

Left ulnar, ADM

ADM = abductor digiti minimi, Amp = amplitude, APB = abductor pollicis brevis, Dist = distance, Lat = latency, Temp = temperature, Vel = velocity

F Wave Studies

Fmin Fmax

(ms) (ms)

Left median, APB 31.15 31.65

Left ulnar, ADM 30.55 33.6

Nerve

ADM = abductor digiti minimi, APB = abductor pollicis brevis

Needle EMG Summary Table

Insertional Fib Fasc Comment Dur Amp Phases Activation

Left deltoid Increased 2+ None CRDS, PSWs 2+ 2+ 1+ Normal

Left biceps Normal None None - N N N Normal

Left triceps Normal None None - N N N NormalLeft teres minor Normal 1+ None PSWs 1+ 1+ 1+ Normal

Left infraspinatus Normal None None - N N N Normal

Left rhomboid major

Normal None None - N N N Normal

Left C5 PSP Normal None None - N N N Normal

Spontaneous MUAP

Recruitment

Slightly reducedNormal

Normal

Normal

Normal

Normal

Normal

Amp = amplitude, CRD = complex repetitive discharges, Dur = duration, Fib = fibrillation, Fasc = fasciculation, PSP = paraspinal, PSW = positive sharp wave

Page 41: NM Update Course II: Inflammatory Myopathies/Myotonic

41

NEUROMUSCULAR UPDATE II

Questions

10A. What is the MOST LIKELY localization of this lesion?

A. Axillary nerve.

B. C5 nerve root.

C. Lateral cord of brachial plexus.

D. Musculocutaneous nerve.

E. Upper trunk of brachial plexus.

10B. Which of the following sensory NCSs would help to distinguish this condition from an upper trunk brachial plexopathy?

A. Lateral antebrachial cutaneous nerve.

B. Medial antebrachial cutaneous nerve.

C. Median nerve, recording at digit III.

D. Ulnar nerve.

E. None of the above.

10C. What is the MOST LIKELY site of nerve entrapment in this case?

A. Arcade of Frohse.

B. Quadrilateral space.

C. Spinoglenoid notch.

D. Suprascapular notch.

E. Thoracic outlet.

VIGNETTE ELEVEN

A 78-year-old man with multiple myeloma undergoing chemo-therapy presents with pain and paresthesias in the hands and feet. Unfortunately, he does not know which chemotherapeutic agent he has received and no medical records are available for review.

Questions

11A. Which of the following chemotherapeutic agents used for mul-tiple myeloma is MOST LIKELY to have produced his symptoms?

A. Bortezomib.

B. Dexamethasone.

C. Lenalidomide.

D. Melphalan.

E. Neuropathy is a common side effect of all the agents listed.

When the patient’s medical records arrive, you learn that he had been given bortezomib to treat his multiple myeloma.

11B. Which of the following is TRUE about the development of poly-neuropathy secondary to this medication?

A. It is an acute-onset neuropathy characterized by paresthe-sias, allodynia, and cold hypersensitivity.

B. Symptoms resolve immediately upon discontinuation of bortezomib.

C. The neuropathy usually presents within the first five cycles of administration.

D. The neuropathy appears several months after completion of chemotherapy.

E. It is difficult to distinguish from POEMS (polyneuropathy, organomegaly, endocrinopathy, monoclonal gammopa-thy, and skin changes) syndrome on electrophysiologic grounds.

11C. Which of the following agents is effective in prophylaxis of bortezomib-induced neuropathy?

A. Calcium.

B. Magnesium.

C. Amitryptiline.

D. Alpha lipoic acid.

E. None of the above.